You are on page 1of 80

Copyright 2014 Delhi Academy of Medical Sciences, All Rights Reserved.

1/80
Test Information
Test Name SWTS-MICROBIOLOGY Total Questions 200

Test Type Examination Difficulty Level Difficult

Total Marks 600 Duration 120minutes

Test Question Language:- ENGLISH

(1). The following is characteristic feature of staphylococcus food poisoning except:

a. Optimum temperature for toxin production is 37C

b. Intradietic toxin are responsible for intestinal symptoms

c. Toxin can be destroyed by boiling for 30 minutes

d. Incubation period is 1-6 hours

Solution. Ans-1: (c) Toxin can be destroyed by boiling for 30 minutes Ref.: Read the text below Sol : Staphylococcal food poisoning :
Staphylococcal food poisoning is due to heat stable preformed toxin mostly after consuming milk products.
Toxin is produced optimally at 35C to 37C
IP- 1.-6 hours.
Mechanism Acts by stimulating vagus nerve and vomiting center of brain

Your Answer. c
Correct Answer. c

(2). Which of the following infestations leads to malabsorption?

a. Giardia lamblia

b. Ascaris lumbricoides

c. Necator Americana

d. Ancylostoma duodenale

Solution. Ans-2.: (a) Giardia lamblia Ref.: Read the text below Sol : Giardia lamblia
Symptoms of infection include (in order of frequency) diarrhea, malaise, excessive gas (often flatulence or a foul or sulphuric-tasting
belch, which has been known to be so nauseating in taste that it can cause the infected person to vomit), steatorrhoea (pale, foul
smelling, greasy stools), epigastric pain, bloating, nausea, diminished interest in food, possible (but rare) vomiting which is often violent,
and weight loss
People with recurring Giardia infections, particularly those with a lack of the immunoglobulin A antibody, may develop chronic disease.
Lactase deficiency may develop in an infection with Giardia, but this usually does not persist for more than a few weeks, and a full
recovery is the norm.
Some studies have shown giardiasis should be considered as a cause of vitamin B12 deficiency as result of the problems caused within
the intestinal absorption system
Ascaris lumbricoides in small intestine usually cause no symptoms. In children it may cause pain and intestinal obstruction sometimes
complicated by perforation, intussusceptions or volvulus.
Migration to aberrant site can cause biliary colic, cholecystitis, cholangitis, pancreatitis or rarely intrahepatic abscess.
Intestinal phase of A. duodenale cause epigastric pain, inflammatory diarrhea and iron deficiency anemia.

Your Answer. a
Correct Answer. a

Copyright 2014 Delhi Academy of Medical Sciences, All Rights Reserved. 2/80
(3). The vector shown below transmits which of the following diseases?

a. Malaria

b. Filaria

c. Yellow fever

d. Kala-azar

Solution. Ans-3: (d) Kala-azar Ref.: Read the text below Sol : Sandfly (Phlebotomus) is the vector of Kala-azar The average life of a
sandfly is about 2 weeks.
Diseases transmitted by Sandfly
SPECIES DISEASES TRANSMITTED

Phlebotomusargentipes Kala-azar

Sandfly fever
Phlebotomuspapatasii
Oriental sore

Phlebotomussergenti Oriental sore

Your Answer. a
Correct Answer. d

(4). Which of the following statements about the varicella vaccine is true?

a. It is indicated in nonimmune pregnant women exposed to varicella

b. It confers lifelong immunity.

c. It can be used to prevent zoster.

d. A single dose between 12 and 18 months is recommended

Solution. Ans-4: (d) A single dose between 12 and 18 months is recommended Ref: Read the text below. Sol:
A single dose between 12 to 18 months is consistent with current CDC recommendations.
Exposed nonimmune pregnant females should be treated with varicella-zoster immune globulin (VZIG).
Data regarding duration of immunity and usefulness against zoster is still lacking. The currently approved vaccine is a live attenuated
strain.

Your Answer. c
Correct Answer. d

Copyright 2014 Delhi Academy of Medical Sciences, All Rights Reserved. 3/80
(5). Coccidioidomycosis is a pathogen in infectious disease. The most common clinical presentation is as

a. Bronchitis

b. Meningitis

c. Osteomyelitis

d. Dermatitis

Solution. Ans-5: (a) Bronchitis Ref: Read the text below. Sol:
C. immitis is a pathogen that is inhaled into the lungs and most commonly presents as a pulmonary infection.
In certain individuals who are at risk, the disease may disseminate and cause a secondary complication of meningitis or osteomyelitis, as
brain and bone are common sites of dissemination.
Dermatitis is a cutaneous manifestation of pulmonary disease

Your Answer. a
Correct Answer. a

(6). Varicella is most likely to occur in which of the following months ?

a. January

b. April

c. June

d. August

Solution. Ans-6: (c) June Ref: Read the text below. Sol:
Chickenpox is an airborne disease spread easily through coughing or sneezing of ill individuals or through direct contact with secretions
from the rash.
A person with chickenpox is infectious from one to five days before the rash appears.
The contagious period continues for 4 to 5 days after the appearance of the rash, or until all lesions have crusted over.
Immunocompromised patients are probably contagious during the entire period new lesions keep appearing. Crusted lesions are not
contagious
Peak incidence of this disorder is late winter or early spring, and specifically the months of March through May.
Why this occurs is unclear, but the pattern is distinct and well known.

Your Answer. d
Correct Answer. c

(7). Which of the following symptoms is usually the first indication of the presence of Lyme disease?

a. Pruritus at the bite site

b. Cellulitis at the bite site

c. Erythema migrans

d. Fever and chills

Solution. Ans-7: (c) Erythema migrans Ref: Read the text below. Sol:
Fully 50 percent of all patients will experience the rash of Lyme disease, erythema migrans.
Stage 1 of the disease is accompanied by the red, raised plaque like annular lesion that enlarges over 3 to 4 weeks with central clearing.
Patients may experience systemic symptoms resembling a viral illness with fever and headache.

Your Answer. d
Correct Answer. c

Copyright 2014 Delhi Academy of Medical Sciences, All Rights Reserved. 4/80
(8). About enterotoxigenic Ecoli true is

a. Does not cause travellers diarrhea

b. Most commonly causes diarhea in deveoping countries

c. Invasion of gut mucosa

d. Fomite borne

Solution. Ans-8: (b) Most commonly causes diarhea in deveoping countries. Ref: Jawetz, Melnick, & Adelberg's Medical Microbiology,
24th ed pg 219. Sol: Enterotoxigenic E coli (ETEC)
It is a common cause of "traveler's diarrhea" and a very important cause of diarrhea in infants in developing countries.
ETEC colonization factors specific for humans promote adherence of ETEC to epithelial cells of the small bowel.
Some strains of ETEC produce a heat-labile exotoxin (LT) (MW 80,000) that is under the genetic control of a plasmid.
Its subunit B attaches to the GM1 ganglioside at the brush border of epithelial cells of the small intestine and facilitates the entry of
subunit A (MW 26,000) into the cell, where the latter activates adenylyl cyclase.
This markedly increases the local concentration of cyclic adenosine monophosphate (cAMP), which results in intense and prolonged
hypersecretion of water and chlorides and inhibits the reabsorption of sodium.
The gut lumen is distended with fluid, and hypermotility and diarrhea ensue, lasting for several days. LT is antigenic and cross-reacts
with the enterotoxin of Vibrio cholerae. LT stimulates the production of neutralizing antibodies in the serum (and perhaps on the gut
surface) of persons previously infected with enterotoxigenic E coli.
Persons residing in areas where such organisms are highly prevalent (eg, in some developing countries) are likely to possess antibodies
and are less prone to develop diarrhea on reexposure to the LT-producing E coli. Assays for LT include the following: (1) fluid
accumulation in the intestine of laboratory animals; (2) typicalcytologic changes in cultured Chinese hamster ovary cells or other cell
lines; (3) stimulation of steroid production in cultured adrenal tumor cells; and (4) binding and immunologic assays with standardized
antisera to LT.
Some strains of ETEC produce the heat-stable enterotoxin STa (MW 15004000), which is under the genetic control of a heterogeneous
group of plasmids. STa activates ganylyl cyclase in enteric epithelial cells and stimulates fluid secretion.
Many STa-positive strains also produce LT. The strains with both toxins produce a more severe diarrhea.
The plasmids carrying the genes for enterotoxins (LT, ST) also may carry genes for the colonization factors that facilitate the attachment
of E coli strains to intestinal epithelium.
Recognized colonization factors occur with particular frequency in some serotypes. Certain serotypes of ETEC occur worldwide; others
have a limited recognized distribution.
It is possible that virtually any E coli may acquire a plasmid encoding for enterotoxins. There is no definite association of ETEC with the
EPEC strains causing diarrhea in children.
Likewise, there is no association between enterotoxigenic strains and those able to invade intestinal epithelial cells.

Your Answer. b
Correct Answer. b

(9). Which component of streptococcal pyogenes has cross reactivity with synovium of human?

a. Capsular hyaluronic acid

b. Cell proteins

c. Group A carbohydrate antigens

d. Peptidoglycan

Solution. Ans-9: (a) Capsular hyaluronic acid Ref.: Read the text below Sol :
Gross Reactivity of streptococcal antigen

Bacterial antigen Cross reacting human component

Human synovial fluid


Capsular hyaluronic acid Cardiac valves
Group A carbohydrate antigen Vascular intima
Cytoplasmic membrane antigen Myocardium
Cell wall protein Skin antigen
Peptidoglycan Sarcolemma of smooth and cardiac muscle, dermal fibroblasts
Membrane antigens And neurons of caudate nucleus

Your Answer. a
Correct Answer. a

Copyright 2014 Delhi Academy of Medical Sciences, All Rights Reserved. 5/80
(10). An HIV positive female has an indurated ulcer over cornmeal agar at 20 degrees, microscopy showing hyphae and growth inhuman
serum at 37 degrees show budding yeasts. The probable cause is:

a. Candida albicans

b. Histoplasmosis

c. Blastomycosis

d. Coccidiodomycosis

Solution. Ans-10: (a) Candida albicans Ref.: Read the text below Sol :
This is a case of oral thrush secondary to candidiasis
C. albicans is a dimorphic fungi which occur both as yeast and moulds (with hyphae)
In HIV oral thrush occurs when CD4 < 50/ul.

Your Answer. a
Correct Answer. a

(11). Regarding HIV which of the following is not true?

a. It is a DNA retrovirus

b. Contains Reverse transcriptase

c. May infect host CD 4 cells other than T lymphoctyes

d. Causes a reduction in host CD 4 cells at late stage of disease

Solution. Ans-11: (a) It is a DNA retrovirus Ref.: Read the text below Sol :
HIV is RNA retrovirus (not DNA virus)
It primarily infect CD 4 + T cells but can infect other cells also which bear CD4 receptor on their surface
These include circulating dendritic cells; epidermal langerhans cells; monocytes.

Your Answer. a
Correct Answer. a

(12). Which of the followings is false about Cl.perfringes

a. Food poisoning strains produce heat resistant spores

b. Show positive naegler reaction

c. Most important toxin is hyaluronidase

d. M.C. cause gas gangrene

Solution. Ans-12: (c) Most important toxin is hyaluronidase . Ref: Ananthanarayan and Panikers textbook of microbiology 8th ed pg 252.
Sol:
Four major toxins produced by clostridia species are alpha, beta, epsilon, iota.
Tha alpha toxin is produced by all strains of clostridia and most predominantly by type A strain
This is the most important toxin biologically and is responsible for profound toxaemia in gas gangrene
Resistance:
Spores are destroyed by boiling with in five minutes but those of food poisoning strains of type A and certain type C strains resists
boiling for 1-3 hours
Naglers reaction
When cl.perfringes is grown on a medium containing 6% agar, 5% fildes peptic digest of sheep blood and 20 % human serum, with the
antitoxin spread on one half of the plate, colonies on the other half with out the antitoxin will be surrounded by a zone of opacity. There
will be no opacity around the colonies on the other half of the plate with the anti toxin due to the specific neutralisation of alpha toxin.
This specific lecithinase effect is known as naglers reaction

Your Answer. a
Correct Answer. c

Copyright 2014 Delhi Academy of Medical Sciences, All Rights Reserved. 6/80
(13). All can cause candidiasis except

a. DM

b. HTN

c. Ca Breast

d. Pregnancy

Solution. Ans-13: (b) HTN. Ref: Harrisons principle of internal medicine 17th ed pg 1254. Sol: Candidiasis have identified the following
predisposing factors or conditions:
a) Antibacterial agents,
b) Indwelling intravascular catheters,
c) Hyperalimentation fluids,
d) Indwelling urinary catheters,
e) Parenteral glucocorticoids,
f) Respirators, neutropenia,
g) Abdominal and thoracic surgery,
h) Cytotoxic chemotherapy, and
i) Immunosuppressive agents for organ transplantation.
j) Patients with severe burns, low-birth-weight neonates, and persons using illicit iv drugs are also susceptible. Hiv-infected patients with
low cd4+ t cell counts and patients with diabetes are susceptible to mucocutaneous infection, which may eventually develop into the
disseminated form when other predisposing factors are encountered.
k) Women who receive antibacterial agents may develop vaginal candidiasis.

Your Answer. b
Correct Answer. b

(14). Amoebiasis is not transmitted through

a. Vertical Transmission

b. Fecooral

c. cockroach

d. Oral

Solution. Ans-14: (a) Vertical Transmission . Ref: Harrisons principle of internal medicine 17th ed pg 1275. Sol: Mode of infection
Fecal oral route: E. histolytica is acquired by ingestion of viable cysts from fecally contaminated water, food, or hands. Food-borne
exposure is most prevalent and is particularly likely when food handlers are shedding cysts or food is being grown with feces-
contaminated soil, fertilizer, or water. Besides the drinking of contaminated water, less common means of transmission include oral and
anal sexual practices andin rare instancesdirect rectal inoculation through colonic irrigation devices. Motile trophozoites are
released from cysts in the small intestine and, in most patients, remain as harmless commensals in the large bowel
Sexual transmission
Vectors such as flies, cockroachs and rodents
1. Boy with skin ulcer on leg. Culture reveals beta hemolysis, Bacitracin sensitive. Cultures from school children with sore throat some
days back also revealed beta hemolysis. What is the similarity between both
a) C carbohydrate antigen is different
b) M protein is same

Your Answer. a
Correct Answer. a

Copyright 2014 Delhi Academy of Medical Sciences, All Rights Reserved. 7/80
(15). Genetic reassortment is found in

a. Hepadna virus

b. Parvo virus

c. Rota virus

d. Astro virus

Solution. Ans-15: (c) Rota virus . Ref: Harrisons principle of internal medicine 17th ed pg 1206. Sol:
Rotaviruses are members of the family Reoviridae.
The viral genome consists of 11 segments of double-strand RNA that are enclosed in a triple-layered, nonenveloped, icosahedral capsid
75 nm in diameter.
h Viral protein 6 (VP6), the major structural protein, is the target of commercial immunoassays and determines the group specificity of
rotaviruses.
h There are seven major groups of rotavirus (A through G); human illness is caused primarily by group A and, to a much lesser extent,
by groups B and C. Two outer-capsid proteins, VP7 (G-protein) and VP4 (P-protein), determine serotype specificity, induce neutralizing
antibodies, and form the basis for binary classification of rotaviruses (G and P types).
h The segmented genome of rotavirus allows genetic reassortment (i.e., exchange of genome segments between viruses) during co-
infectionXa property that may play a role in viral evolution and has been utilized in the development of reassortant animal-human
rotavirusVbased vaccines.

Your Answer. c
Correct Answer. c

(16). A normally healthy person who has resided in the agricultural area presents with complaints of fever, malaise, cough, and chest pain.
The most likely differential for this illness would be

a. Histoplasmosis

b. Blastomycosis

c. Coccidioidomycosis

d. Aspergillosis

Solution. Ans-16: (c) Coccidioidomycosis Ref: Read the text below. Sol:
Coccidioides immittis is the fungus that resides in that geographic area.
Healthy persons new to the area are most likely to contract the disease in the first 3 years of residency.
Aspergillosis is more commonly found in patients that have < 500 granulocyte count, have exposure to cytotoxic drugs, or are on large
amounts of glucocorticoids.

Your Answer. a
Correct Answer. c

Copyright 2014 Delhi Academy of Medical Sciences, All Rights Reserved. 8/80
(17). Which of the following is true regarding diphtheria ?

a. The pseudomembrane extending to the bronchi is usually fatal.

b. Probable case that is not laboratory-confirmed and is epidemiologically linked to a laboratory-confirmed case

c. Bull neck refers to neck muscle spasms.

d. Diphtheria has a long incubation period

Solution. Ans-17: (a) The pseudomembrane extending to the bronchi is usually fatal. Ref: Read the text below. Sol:
The pseudomembrane of diphtheria, when it extends to the bronchi, is usually fatal. The other responses are incorrect statements.
The current definition of diphtheria used by the Centers for Disease Control and Prevention (CDC) is based on both laboratory and
clinical criteria. Laboratory criteria
Isolation of Corynebacterium diphtheriae from a clinical specimen, or
Histopathologic diagnosis of diphtheria
Clinical criteria
Upper respiratory tract illness with sore throat
Low-grade fever (>103F is rare)
An adherent pseudomembrane of the tonsil(s), pharynx, and/or nose.
Case classification
Probable: a clinically compatible case that is not laboratory-confirmed and is not epidemiologically linked to a laboratory-confirmed case
Confirmed: a clinically compatible case that is either laboratory-confirmed or epidemiologically linked to a laboratory-confirmed case

Your Answer. a
Correct Answer. a

(18). Unvaccinated diphtheria carriers should be treated with

a. Antitoxin

b. Antitoxin and antibiotics

c. Diphtheria toxoid vaccination

d. Diphtheria toxoid vaccination and antibiotics

Solution. Ans-18: (d) Diphtheria toxoid vaccination and antibiotics Ref: Read the text below. Sol:
It is recommended that diphtheria carriers be treated both with antibiotics and the diphtheria toxoid vaccination.

Your Answer. d
Correct Answer. d

(19). Which of the following are the most common fatal complications of measles?

a. Fever and bacterial pneumonia

b. Hepatitis and giant cell pneumonia

c. Meningitis and respiratory complications

d. SSPE and immunodeficiency

Solution. Ans-19: (C) Meningitis and respiratory complications Ref: Read the text below. Sol:
Complications with measles are relatively common, ranging from relatively mild and less serious diarrhea, to pneumonia and acute
encephalitis (and rarely subacute sclerosing panencephalitis); corneal ulceration leading to corneal scarring.
Complications are usually more severe amongst adults who catch the virus.
The most common fatal complications of measles are meningitis and respiratory complications, such as pneumonia.

Your Answer. d
Correct Answer. c

Copyright 2014 Delhi Academy of Medical Sciences, All Rights Reserved. 9/80
(20). Which of the following is not a contraindication for mumps vaccination?

a. Young woman trying to get pregnant

b. Upper respiratory infection without fever

c. Pregnancy

d. Allergy to egg products

Solution. Ans-20: (b) Upper respiratory infection without fever Ref: Read the text below. Sol:
A mild nonfebrile upper respiratory infection is not a complication for immunization.
The other statements are contraindications to the mumps vaccine.

Your Answer. a
Correct Answer. b

(21). In hospital ward MRSA spread is stopped by

a. Fumigating room

b. Proper hand washing

c. Ethylene Oxide

d. None

Solution. Ans-21: (b) Proper hand washing. Ref: Harrisons principle of internal medicine 17th ed pg 878. Sol:
Prevention of the spread of S. aureus infections in the hospital setting involves hand washing and careful attention to appropriate
isolation procedures

Your Answer. b
Correct Answer. b

(22). Malignant pustule is caused by

a. Carbuncle

b. Ulcerating melanoma

c. Anthrax of skin

d. CMV

Solution. Ans-22: (c) Anthrax of skin Ref: Ananthanarayan and Panikers textbook of microbiology 8th ed pg 244, Jawetz, Melnick, &
Adelberg's Medical Microbiology, 24th ed pg 166. Sol:
Cutaneous anthrax generally occurs on exposed surfaces of the arms or hands, followed in frequency by the face and neck.
A pruritic papule develops 17 days after entry of the organisms or spores through a scratch.
Initially it resembles an insect bite. The papule rapidly changes into a vesicle or small ring of vesicles that coalesce, and a necrotic ulcer
develops.
This lesion is known as malignant pustule
The lesions typically are 13 cm in diameter and have a characteristic central black eschar.
Marked edema occurs. Lymphangitis and lymphadenopathy and systemic signs and symptoms of fever, malaise, and headache may
occur. After 710 days the eschar is fully developed.
Eventually it dries, loosens, and separates; healing is by granulation and leaves a scar. It may take many weeks for the lesion to heal and
the edema to subside.
Antibiotic therapy does not appear to change the natural progression of the disease. In as many as 20% of patients, cutaneous anthrax
can lead to sepsis, the consequences of systemic infectionincluding meningitisand death.

Your Answer. c
Correct Answer. c

Copyright 2014 Delhi Academy of Medical Sciences, All Rights Reserved. 10/80
(23). A 2-year-old infant is brought to the emergency room with hemolytic uremic syndrome and thrombocytopenia. Which one of the following
bacteria would most likely be isolated from a stool specimen?

a. Shigellab

b. Salmonella

c. Aeromonas

d. E. coli 0157/H7

Solution. Ans 23 : (d) E. coli 0157/H7 Ref:Read the text below Sol:
Food poisoning with E. coli 0157/H7 causes hemorrhagic colitis; it is often seen after eating beef hamburgers.
The same organism also causes a hemorrhagic uremic syndrome. The toxin, called Shiga-like toxin, can be demonstrated in Vero cells,
but the cytotoxicity must be neutralized with specific antiserum.
With the exception of sorbitol fermentation, there is nothing biochemically distinctive about these organisms.

Your Answer. d
Correct Answer. d

(24). Infection with Sporothrix schenckii (formerly Sporotrichum schenckii) is an occupational hazard for gardeners. Which of the following is
the portal of entry for this organism?

a. Lymphatic system

b. Respiratory tract

c. Skin

d. Mouth

Solution. Ans 24 : (c) Skin Ref:Read the text below Sol:


Cutaneous sporotrichosis, caused by S. schenckii, begins at the site of inoculation, usually on an extremity or the face.
The organism often is found on thorns of rose bushes.
Ulceration is common and new lesions appear along paths of lymphatic channels. Extracutaneous sporotrichosis is seen primarily in
bones and joints.
There is no evidence to suggest that any portal of entry besides skin is important

Your Answer. c
Correct Answer. c

(25). C. albicans is recognized in microscopic examination of infected tissues by the presence of which of the following?

a. Spherules containing endospores

b. Metachromatic granules

c. Yeasts and pseudohyphae

d. Asci containing 28 ascospores

Solution. Ans 25 : (c) Yeasts and pseudohyphae Ref:Read the text below Sol:
C. albicans is the most important species of Candida and causes thrush, vaginitis, skin and nail infections, and other infections.
It is part of the normal flora of skin, mouth, GI tract, and vagina. It appears in tissues as an oval budding yeast or elongated
pseudohyphae. It grows well on laboratory media and is identified by germ-tube formation.
A vaccine is not available and serologic and skin tests have little value

Your Answer. b
Correct Answer. c

Copyright 2014 Delhi Academy of Medical Sciences, All Rights Reserved. 11/80
(26). Which toxin of streptococcus causes hemolysis

a. Streptolysin O

b. Streptolysin S

c. Streptokinase

d. Streptodornase

Solution. Ans-26: (b) Streptolysin S Ref.: Read the text below Sol :
Streptococci Produce two hemolysin

Streptolysin O Streptolysin S (Serum soluble)

Oxygen labile Oxygen stable


Activity only on pour plate not on surface Responsible for hemolysis seen around streptococcal colonies on surface
Antigenic protein Non antigenic protein elaborated in presence of serum

Your Answer. a
Correct Answer. b

(27). A patient admitted to an ICU is on central venous line for the last one week. He is on ceftazidime and amikacin. After 7 days of antibiotics
he develops a spike of fever and his blood cultyure is positive for gram positive cocci in chains which are catalase-negative. Following
this vancomycin was started but the culture remained positive for same organism even after 2 weeks of therapy. The most likely
organism causing infection is:

a. Staphylococci aureus

b. Viridans streptococci

c. Enterococcus faecalis

d. Coagulase negative staphylococcus

Solution. Ans-27: (c) Enterococcus faecalis Ref.: Read the text below Sol :
Enterococci are catalase negative and grow in chains and above all resistant to cephalosporins.
Enterococci is a frequent cause of nosocomial bacteremias and many of these enterococci are resistant to vancomycin.
Enterococcal bacteremias is characteristically seen in ICU in patient taking cephalosporin as antibiotic.

Your Answer. b
Correct Answer. c

(28). Which infection is not common in HIV patients?

a. Cryptosporidiosis

b. Atypical mycobacterial infection

c. Aspergillosis

d. Candidiasis

Solution. Ans-28: (c) Aspergillosis Ref.: Read the text below Sol :
All of the following organism can cause opportunistic infection in AIDS patient.
But invasive aspergillosis is generally not seen in patient with AIDS in the absence of neutropenla or administration of glucocorticoids.
Aspergillosis also given in list of opportunistic infection but is not much common, hence is best option.

Your Answer. c
Correct Answer. c

Copyright 2014 Delhi Academy of Medical Sciences, All Rights Reserved. 12/80
(29). A 30 years old patient presented with history of jaundice for 10 days. His liver function tests showed bilirubin of 10 mg/dl, SGOT/SGPT
100/1450, serum alkaline phosphatase-240 IU. He was positive for HbsAg. What sould be the confirmatory test to establish acute
hepatitis B infection?

a. IgM Anti-HBc antibody

b. HbeAg

c. HBV DNA by PCR

d. Anti-HBc antibody

Solution. Ans-29: (a) IgM Anti-HBc antibody Ref.: Read the text below Sol :
Test diagnostic of acute infection IgM anti HBc antibody
Test diagnostic of chronic infection IgG anti-HBc + HBsAg
Test for determining infectivity HbeAg.

Your Answer. a
Correct Answer. a

(30). Transmission of hepatitis A virus occurs :

a. One week before and one week after onset of symptom

b. 4 weeks before onset of symptom

c. 2 week after onset of symptom

d. 1 week after onset of symptom

Solution. Ans-30: (d) 1 week after onset of symptom Ref.: Read the text below Sol :
The risk of transmitting HAV is greatest from 2 weeks before to 1 week after the onset of Jaundice

Your Answer. b
Correct Answer. d

(31). The sputum specimen of a 70 year old male was cultured on a 5% blood agar. The culture showed the presence of hemolytic colonies next
day. The further processing of this organism is most likely to yield :

a. Gram-positive cocci in short chains, catalase negative and bile resistant.

b. Gram-positive cocci in pairs, catalase negative and bile soluble

c. Gram-positive cocci in clusters, catalase positive and coagulase positive

d. Gram-negative coccobacilli, catalase positive and oxidase positivea

Solution. Ans-31: (b) Gram-positive cocci in pairs, catalase negative and bile soluble Ref.: Read the text below Sol :
Bacteria which sows
or partial hemolysis includes :

Your Answer. b
Correct Answer. b

Copyright 2014 Delhi Academy of Medical Sciences, All Rights Reserved. 13/80
(32). Vaccine preparation requires which virus as vector:

a. Rhinovirus

b. Vaccinia

c. Adenovirus

d. Hepatitis B

Solution. Ans-32: (b) Vaccinia Ref.: Read the text below Sol :
Vaccinia virus is unique in that it is an artificial virus and does not occur in nature as such.
It is used as a vector for development of recombinant vaccines.
Its genome can accommodate 25,000 foreign base pairs.
Genes encoding antigens of HBV, HIV, rabies and neuropeptides are inserted in it.
However it is not useful as vector for human use due to pathogenic effect.

Your Answer. b
Correct Answer. b

(33). The following statements are true regarding leptospirosis except :

a. It is a zoonosis

b. Man is the dead end host

c. Man is an accidental host

d. Lice acts as reservoir of infection

Solution. Ans-33: (d) Lice acts as reservoir of infection Ref.: Read the text below Sol :
Reservoir of leptospirosis are rats not lice.
Transmission occur through direct contact Leptospira enter the body through skin abrasions or through intact mucous membrane :
Ingestion of food and water contaminated with leptospria.
Inhalation of droplets of urine of infected animal.

Your Answer. d
Correct Answer. d

(34). Which of the following statement regarding T. pallidum is incorrect;

a. Can be maintained in rabbit testis

b. Motile by peritrichate flagella

c. To visualise, dark ground microscopy is used

d. TPI test is very useful

Solution. Ans-34: (b) Motile by peritrichate flagella Ref.: Read the text below Sol : T.pallidum
Motile by endolflagella not peritirichate flagella.
Endoflagella are not visible outside.
Visualized on dark ground microscopy.
Pathogenic treponemes do not grow in artificial culture media.
Can be maintained in rabbit testis
Have rotational movement, forward backward movement, flexion of whole body.

Your Answer. a
Correct Answer. b

Copyright 2014 Delhi Academy of Medical Sciences, All Rights Reserved. 14/80
(35). Inhalation of fungal spores can cause primary lung infections. Of the following organisms, which one is most likely to be associated with
this mode of
transmission?

a. C. immitis

b. S. schenckii

c. C. albicans

d. T. tonsurans

Solution. Ans 35: (a) C. immitis Ref:Read the text below Sol:
C. albicans and Candida tropicalis are opportunistic fungi, and as part of the normal flora are not transmitted by inhalation.
C. immitis is a dimorphic fungus and inhalation of the spores transmits the infection. Sprothrix is also a dimorphic fungus but its portal
of entry is cutaneous.
Trychophyton is a dermatophyte and one of the causes of athlete's foot

Your Answer. a
Correct Answer. a

(36). Tinea cruris is caused by which of the following?

a. E. floccosum

b. M. furfur

c. M. canis

d. E. werneckii

Solution. Ans 36 : (a) E. floccosum Ref:Read the text below Sol:


Dermatomycoses are cutaneous mycoses caused by three genera of fungi: Microsporum, Trichophyton, and Epidermophyton.
These infections are called tinea or ringworm, a misnomer that has persisted from the days when they were thought to be caused by
worms or lice.
Tinea corporis (ringworm of the body) is caused by M. canis and T. mentagrophytes.
This disorder affects smooth skin and produces circular pruritic areas of redness and scaling. Both tinea cruris (ringworm of the groin,
"jock itch") and tinea pedis (ringworm of the feet, athlete's foot) are caused by T. rubrum, T. mentagrophytes, or E. floccosum.
These common conditions are pruritic and can cause scaling.

Your Answer. a
Correct Answer. a

(37). One of the most common sexually transmitted diseases that may lead to cervical carcinoma is caused by which of the following viruses?

a. Cytomegalovirus

b. Papillomavirus

c. Epstein-Barr virus

d. Herpes simplex virus

Solution. Ans 37 : (b) Papillomavirus Ref:Read the text below Sol:


Human papillomavirus (HPV) is the cause of genital warts. It is one of the most pervasive of all the sexually transmitted diseases.
There is no specific cure or vaccine. There are multiple serotypes of papillomavirus and some serotypes are linked to cervical cancer.
New techniques for molecular diagnosis of HPV show promise for rapid and sensitive detection and perhaps more aggressive treatment.

Your Answer. b
Correct Answer. b

Copyright 2014 Delhi Academy of Medical Sciences, All Rights Reserved. 15/80
(38). The following diagram illustrates the amplification of DNA by the polymerase chain reaction (PCR). There are at least four points in the
PCR process (AD) that are critical to the reaction.

The Southern blot detection system for amplified PCR products fails to function. Which one of the following would be affected?

a. A

b. B

c. C

d. D

Solution. Ans 38: (d) D Ref :Read the text below Sol:
The process of PCR is complicated and its steps are interrelated.
A number of steps in the process can markedly affect the results of clinical testing.
For example, the detection of amplified products is essential in order to determine whether target nucleic acid was present in the
specimen.
Product can be detected by staining of the gel that separates the products, Southern blot (a radioactive procedure), or an ELISA-like
capture method.
A failure of this production step prevents detection of product

Your Answer. a
Correct Answer. d

(39). Kuru is a fatal disease and is characterized by tremors and ataxia; Creutzfeldt-Jakob disease (CJD) is characterized by both ataxia and
dementia. These diseases are thought to be caused by which of the following?

a. Slow viruses

b. Cell walldeficient bacteria

c. Environmental toxins

d. Prions

Solution. Ans 39 : (d) Prions Ref:Read the text below Sol:


Kuru and Creutzfeldt-Jakob disease (CJD) are similar but not identical diseases with very different epidemiology.
CJD is found worldwide and has been transmitted by corneal transplants and in pituitary hormone preparations.
There is some association between CJD and Mad Cow Disease in England. Prions are unconventional selfreplicating proteins, sometimes
called amyloid.
It is now thought that CJD, Kuru, and animal diseases such as scrapie, visna, and bovine spongiform encephalopathy (Mad Cow Disease)
are caused by prions.

Your Answer. d
Correct Answer. d

Copyright 2014 Delhi Academy of Medical Sciences, All Rights Reserved. 16/80
(40). Which of the following organisms may be ingested with raw fish, affects the liver, and has an operculated egg?

a. Paragonimus

b. Clonorchis

c. S. mansoni

d. S. japonicum

Solution. Ans 40 : (b) Clonorchis Ref:Read the text below Sol:


The life cycle of the medically important trematodes (or flukes) involves a sexual cycle in humans and an asexual cycle in snails. The
schistosomes can penetrate the skin whereas Clonorchis and Paragonimus are ingested, usually in fish or seafood.

Your Answer. d
Correct Answer. b

(41). G. lamblia is best diagnosed by which of the following?

a. Sigmoidoscopy and aspiration of mucosal lesions

b. Baermann technique

c. Dilution followed by egg count

d. Enzyme immunoassay (EIA)

Solution. Ans 41 : (d) Enzyme immunoassay (EIA) Ref:Read the text below Sol:
The diagnosis of giardiasis is usually made by detecting trophozoites and cysts of G. lamblia in consecutive fecal specimens.
Alternatively, a gelatin capsule on a string (enterotest) can be swallowed, passed to the duodenum, and then retrieved after 4 h. The
string is then examined for Giardia. A recent innovation is the introduction of an enzyme immunoassay (EIA) for G. lamblia.
The EIA is more sensitive than microscopy, can be performed on a single stool specimen, and does not depend on the presence of entire
trophozoites and cysts.

Your Answer. d
Correct Answer. d

(42). A latent, measles-like viral infection and, presumably, a defect in cellular immunity is associated with which of the following diseases?

a. Progressive multifocal leukoencephalopathy (PML)

b. Multiple sclerosis (MS)

c. Creutzfeldt-Jakob disease

d. Subacute sclerosing panencephalitis (SSPE)

Solution. Ans 42 : (d) Subacute sclerosing panencephalitis (SSPE) Ref Read the text below Sol:
Measles-like virus has been isolated from the brain cells of patients with subacute sclerosing panencephalitis (SSPE).
The role of the host immune response in the causation of SSPE has been supported by several findings including the following:
(1) progression of disease despite high levels of humoral antibody; (2) presence of a factor that blocks lymphocyte-mediated immunity to
SSPE-measles virus in SSPE cerebrospinal fluid (CSF); (3) lysis of brain cells from SSPE patients by SSPE serum or CSF in the presence
of complement (a similar mechanism could cause in vivo tissue injury).

Your Answer. d
Correct Answer. d

Copyright 2014 Delhi Academy of Medical Sciences, All Rights Reserved. 17/80
(43). During plasma sterilization testing which of the following is used for detecting efficacy of sterilization

a. Bacillus subtilis

b. Bacillus stearothermophilus

c. Cl tetani

d. All of above

Solution. Ans-43: (b) Bacillus stearothermophilus . Ref: Ananthanarayan and Panikers textbook of microbiology 8th ed pg 32. Sol:
Plasma is sterilized by autoclaving
Sterilization control for determining the efficacy of autoclaving is done by spores of Bacillus Stearothermophilus. It is a thermophillic
organism with optimumgrowth temperature of 55-600c and its spores require exposure of 12 minutes at 1210c to be killed
Also know
Spores of non toxigenic strain of Clostridium Tetani are used for microbiological testing of dry heat efficiency.

Your Answer. b
Correct Answer. b

(44). All are true about Heamophillus influenza except

a. Requires factors x and v

b. Doesnt affect infants below 2 months of age

c. Capsular polypeptide protein is responsible for virulence

d. M.C. invasive disease of H influenza is meningitis

Solution. Ans-44: (c) Capsular polypeptide protein is responsible for virulence. Ref: Jawetz, Melnick, & Adelberg's Medical Microbiology,
24th ed pg 245. Sol: Haemophilus influenza
The organisms are short (1.5 micro m) coccoid bacilli, sometimes occurring in pairs or short chains.
In cultures, the morphology depends both on age and on the medium. At 68 hours in rich medium, the small coccobacillary forms
predominate.
Later there are longer rods, lysed bacteria, and very pleomorphic forms.
On chocolate agar, flat, grayish-brown colonies with diameters of 12 mm are present after 24 hours of incubation.
IsoVitaleX in media enhances growth. H influenzae does not grow on sheep blood agar except around colonies of staphylococci ("satellite
phenomenon
Identification of organisms of the haemophilus group depends in part upon demonstrating the need for certain growth factors called X
and V.
Factor X acts physiologically as hemin; factor V can be replaced by nicotinamide adenine nucleotide (NAD) or other coenzymes.
Colonies of staphylococci on sheep blood agar cause the release of NAD, yielding the satellite growth phenomenon.
Carbohydrates are fermented poorly and irregularly.
Encapsulated H influenzae contains capsular polysaccharides (MW > 150,000) of one of six types (af).
The capsular antigen of type b is a polyribose-ribitol phosphate (PRP).
Encapsulated H influenzae can be typed by slide agglutination, coagglutination with staphylococci, or agglutination of latex particles
coated with type-specific antibodies.
A capsule swelling test with specific antiserum is analogous to the quellung test for pneumococci.
Typing can also be done by immunofluorescence. Most H influenzae organisms in the normal flora of the upper respiratory tract are not
encapsulated.
Infants under age 3 months may have serum antibodies transmitted from the mother. During this time H influenzae infection is rare, but
subsequently the antibodies are lost.
Children often acquire H influenzae infections, which are usually asymptomatic but may be in the form of respiratory disease or
meningitis. H influenzae has been the most common cause of bacterial meningitis in children from 5 months to 5 years of age.
By age 35 years, many unimmunized children have naturally acquired anti-PRP antibodies that promote complement-dependent
bactericidal killing and phagocytosis.
Immunization of children with H influenzae type b conjugate vaccine induces the same antibodies.

Your Answer. b
Correct Answer. c

Copyright 2014 Delhi Academy of Medical Sciences, All Rights Reserved. 18/80
(45). Vi polysaccharide true is

a. Can be given with yellow fever in hepatitis

b. Produce severe local reaction

c. Produce many adverse effects

d. Has many contraindications

Solution. Ans-45: (a) Can be given with yellow fever in hepatitis Ref: Read the text below. Sol:
h Typhoid Vi Polysaccharide Vaccine, produced by Sanofi Pasteur SA, for intramuscular use, is a sterile solution containing the cell
surface Vi polysaccharide extracted from Salmonella enterica serovar Typhi, S typhi Ty2 strain.
h The organism is grown in a semi-synthetic medium without animal proteins.
h The capsular polysaccharide is precipitated from the concentrated culture supernatant by the addition of
hexadecyltrimethylammonium bromide and the product is purified by differential centrifugation and precipitation.
h The potency of the purified polysaccharide is assessed by molecular size and O-acetyl content.
h Can be given with yellow fever in hepatitis

Your Answer. a
Correct Answer. a

(46). Which of the following diseases is not transmitted by the vector shown in image below?

a. West Nile fever

b. Rift Valley fever

c. Chikungunya fever

d. Yellow fever

Solution. Ans-46: (a) West Nile fever Ref.: Read the text below Sol :
The vector shown is Aedesaegypti.West Nile fever is transmitted by Culex.
Aedesaegypti occupies a very special position in preventive medicine.It is the first proved vector of a virus disease-Yellow fever.

Your Answer. d
Correct Answer. a

Copyright 2014 Delhi Academy of Medical Sciences, All Rights Reserved. 19/80
(47). All are true regarding tetanus except

a. Transmission through contaminated wounds and injuries

b. More common in winters in dry weather

c. Reservoir in soil n intestines of humans n animals

d. No herd immunity or life long immunity

Solution. Ans-47: (b) More common in winters in dry weather . Ref: Park 20th ed pg 97, Jawetz, Melnick, & Adelberg's Medical
Microbiology, 24th ed pg 168. Sol: Clostridium tetani
Clostridium tetani, which causes tetanus, is worldwide in distribution in the soil and in the feces of horses and other animals.
C tetani is not an invasive organism.
The infection remains strictly localized in the area of devitalized tissue (wound, burn, injury, umbilical stump, surgical suture) into which
the spores have been introduced.
The volume of infected tissue is small, and the disease is almost entirely a toxemia. Germination of the spore and development of
vegetative organisms that produce toxin are aided by (1) necrotic tissue, (2) calcium salts, and (3) associated pyogenic infections, all of
which aid establishment of low oxidation-reduction potential.
About Herd immunity Park writes
It is the level of resistance of a community or a group of people to a particular disease
Factors which contribute to herd immunity
a) Occurance of clinical or subclinical infection in the herd
b) Immunisation of the herd
c) Herd structure
In case of tetanus herd immunity does not protect the community

Your Answer. b
Correct Answer. b

(48). All of the following are true regarding legionella except :

a. Legionella can be grown on complex media

b. L. pneumophilia serogroup 1 is the most common serogroup isolated from humans

c. Legionella are communicable from infected patient to others

d. L. pneumophila is not effectively killed by polymorphonuclear leukocyte

Solution. Ans-48: (c) Legionella are communicable from infected patient to others Ref.: Read the text below Sol :
There is no man-to-man transmission.
Modes of transmission are aerosolization, aspiration and direct instillation of contaminated water into the lung.
L. pneumophilia is most common legionella causing human disease.
Serogroup 1, 4 and 6 are most commonly implicated.
Cell mediated immunity is primary mechanism of host defence.
Alveolar macrophage readily phagocytose legionella; many are killed but some proliferate intracellularly until cell rupture.

Your Answer. c
Correct Answer. c

Copyright 2014 Delhi Academy of Medical Sciences, All Rights Reserved. 20/80
(49). Brucella is transmitted by all the following means except :

a. Through placenta of animals

b. Person to person transmission

c. Aerosol

d. Eating uncooked food

Solution. Ans-49: (b) Person to person transmission Ref.: Read the text below Sol :
Modes of Transmission of Brucella
Contact infection (MC) Food borne infection Airborne infection

Direct contact with infected tissue, blood, urine, Ingestion of raw milk or dairy products
etc. From aerosols in cowshed and slaughter
Water contaminated with excreta of infected house
Mostly occupational animal

Your Answer. d
Correct Answer. b

(50). An organism grown on agar shows green colored colonies; likely organism is :

a. Staphylococcus

b. E. coli

c. Pseudomonas

d. Peptostreptococcus

Solution. Ans-50: (c) Pseudomonas Ref.: Read the text below Sol : Pseudomonas Aerobic, nonsporing gram-negative, motile bacilli, forms
many pigments :
Pyocyanin : Bluish green pigment, produced only by Ps. Aeruginosa
Fluorescin (Pyoverdin) : Greenish yellow pigment which oxidize in old culture to yellowish brown pigment
Pyrubin Red
Pyomelanin Brown

Your Answer. c
Correct Answer. c

(51). True about cholera vibrios is:

a. Can tolerate wide range of alkaline pH

b. Nonmotile bacilli

c. Cant be grown in media

d. NaCI stimulates growth

Solution. Ans-51: (a) Can tolerate wide range of alkaline pH Ref.: Read the text below Sol :
Vibrio cholerae can grow in pH range 6.4-9.6 (Optimum 8.2).
NaCI (0.5-1%) is required for optimal growth through high concentration (6% and above) is inhibitory.
Can be grown on media.
Motile by single polar flagellum.

Your Answer. b
Correct Answer. a

Copyright 2014 Delhi Academy of Medical Sciences, All Rights Reserved. 21/80
(52). Streptococcal toxic shock syndrome is due to the following virulence factor :

a. M-protein

b. Pyrogenic exotoxin

c. Streptolysin O

d. Carbohydrate cell wall

Solution. Ans-52: (b) Pyrogenic exotoxin Ref.: Read the text below Sol :
Most common cause of toxic shock syndrome staphylococci
Most common cause of streptococcal TSS Pyrogenic exotoxin A = erythrogenic /Dick/Scarlatinal toxin.
Most common cause of staphylococcal TSS TSST = Pyrogenic exotoxin C = Enterotoxin type F.

Your Answer. b
Correct Answer. b

(53). Selective media for vibrio:

a. TCBS

b. Stuart

c. Skirrows

d. MYPA

Solution. Ans-53: (a) TCBS Ref.: Read the text below Sol :
TCBS (media containing thiosulphate, citrate, bilesalts, sucrose) is the best selective media for vibrio.
Vibrio produces yellow convex colonies.

Your Answer. a
Correct Answer. a

(54). The following are gas producing Salmonella except:

a. S. typhi

b. S. enteritidis

c. S. cholera

d. S. typhimurium

Solution. Ans-54: (a) S. typhi Ref.: Read the text below Sol :
All Salmonella except S. typhi ferment glucose, mannitol and maltose, forming acid and gas;
S. typhi is anaerogenic.

Your Answer. c
Correct Answer. a

Copyright 2014 Delhi Academy of Medical Sciences, All Rights Reserved. 22/80
(55). 35 year old labourer 3 year back presented with single indurated penile ulcer not treated. Later he presented with skin rash with
neurological symptoms for which he got treated. Test to moniter response to treatment is

a. TPI

b. VDRL

c. FTA-ABS

d. Dark field microscopy

Solution. Ans-55: (b) VDRL . Ref: Jawetz, Melnick, & Adelberg's Medical Microbiology, 24th ed pg 303. Sol: Serologic Tests for Syphilis
(STS)
These tests use either nontreponemal or treponemal antigens.
Nontreponemal Antigen Tests
The antigens employed are lipids extracted from normal mammalian tissue.
The purified cardiolipin from beef heart is a diphosphatidylglycerol. Lecithin and cholesterol are added to enhance reaction with
syphilitic "reagin" antibodies.
Reagin is a mixture of IgM and IgG antibodies directed against the cardiolipin-cholesterol-lecithin complex.
The VDRL (Venereal Disease Research Laboratory) and RPR (rapid plasma reagin) tests are nontreponemal antigen tests used most
commonly.
The toluidine red unheated serum test (TRUST) also is available.
All of the tests are based on the fact that the particles of the lipid antigen remain dispersed with normal serum but flocculate when
combining with reagin.
The VDRL test requires microscopic examination to detect flocculation, whereas the RPR and TRUST have added colored particles and
can be read without microscopic magnification.
Results develop within a few minutes, particularly if the suspension is agitated. The tests lend themselves to automation and to use for
surveys because of their low cost.
Positive VDRL or RPR tests develop after 23 weeks of untreated syphilitic infection and are positive in high titer in secondary syphilis.
Positive VDRL or RPR tests revert to negative in 618 months after effective treatment of syphilis.
The VDRL test can also be performed on spinal fluid and becomes positive after 48 weeks of infection.
Reagin antibodies do not reach the cerebrospinal fluid from the bloodstream but are probably formed in the central nervous system in
response to syphilitic infection.
The flocculation tests can give quantitative results.
An estimate of the amount of reagin present in serum can be made by performing the tests with twofold dilutions of serum and
expressing the titer as the highest dilution that gives a positive result.
Quantitative results are valuable in establishing a diagnosisespecially in neonatesand in evaluating the effect of treatment.
Nontreponemal tests are subject to "biologic" false-positive results attributable to the occurrence of "reagins" in a variety of human
disorders. Prominent among them are other infections (malaria, leprosy, measles, infectious mononucleosis, etc), vaccinations, collagen-
vascular diseases (systemic lupus erythematosus, polyarteritis nodosa, rheumatic disorders), and other conditions.
Treponemal Antibody Tests
Fluorescent Treponemal Antibody (FTA-ABS) Test
This is a test employing indirect immunofluorescence (killed T pallidum + patient's serum + labeled antihuman gamma globulin).
It shows excellent specificity and sensitivity for syphilis antibodies if the patient's serum has been absorbed with sonicated Reiter
spirochetes prior to the FTA test.
The FTA-ABS test is the first to become positive in early syphilis, is routinely positive in secondary syphilis, and usually remains positive
many years after effective treatment.
The test thus cannot be used to judge the efficacy of treatment. The presence of IgM FTA in the blood of newborns is good evidence of in
utero infection (congenital syphilis).
Treponema pallidum-Particle Agglutination (TP-PA) Test
These are the T pallidum hemagglutination (TPHA) and microhemagglutination for T pallidum(MHA-TP) tests. Particles are sensitized
with T pallidum subspecies pallidum antigens.
The test is performed with diluted serum. Antibodies against T pallidum react with the sensitized particles.
A mat of agglutinated particles indicates a positive result. These tests are similar to the FTA-ABS test in specificity and sensitivity.

Your Answer. b
Correct Answer. b

Copyright 2014 Delhi Academy of Medical Sciences, All Rights Reserved. 23/80
(56). The etiologic agent of granuloma inguinale is

a. Haemophilus ducreyi

b. Leishmania donovani

c. Calymmatobacterium granulomatis

d. Leptospira canicola

Solution. Ans-56: (c) Calymmatobacterium granulomatis Ref: Read the text below. Sol:
Calymmatobacterium granulomatis is the etiologic agent of granuloma inguinale.
Small, painless nodules appear after about 1040 days of the contact with the bacteria. Later the nodules burst, creating open, fleshy,
oozing lesions. The infection spreads, mutilating the infected tissue.
The infection will continue to destroy the tissue until treated. The lesions occur at the region of contact typically found on the shaft of
the penis, the labia, or the perineum.
Rarely, the vaginal wall or cervix is the site of the lesion. At least one case in India led to partial auto-amputation of the penis. The
patient tested positive for HIV-2 and had been infected for six years

Your Answer. c
Correct Answer. c

(57). At which stage of syphilis is the patient likely to have condylomata lata ?

a. Primary syphilis

b. Secondary syphilis

c. Latent syphilis

d. Tertiary syphilis

Solution. Ans-57: (b) Secondary syphilis Ref: Read the text below. Sol:
Condylomata lata or condyloma latum, is a cutaneous condition characterized by wart like lesions on the genitals.
They are generally symptoms of the secondary phase of syphilis, caused by the spirochete, Treponema pallidum.
Condylomata lata occurs in about one third of patients and is characterized by painless, mucosal, warty erosions. They tend to develop in
warm, moist sites of the genitals and perineum.
These lesions hold a high accumulation of spirochetes and are highly infectious.
Complete resolution of the lesions is spontaneous and occurs after a few days to many weeks where it is either resolved completely or
enters the tertiary phase, defined by a latent state

Your Answer. b
Correct Answer. b

(58). The acute HIV syndrome most closely resembles

a. Influenza infection

b. Mononucleosis

c. Herpes zoster

d. Pityriasis rosea

Solution. Ans-58: (b) Mononucleosis Ref: Read the text below. Sol: The acute HIV syndrome consists of lymphadenopathy with fever,
pharyngitis, arthralgias, myalgias, headache, lethargy, and anorexia.
It most closely resembles mononucleosis and, because of its perceived benign nature in presentation, is often missed or not reported.

Your Answer. a
Correct Answer. b

Copyright 2014 Delhi Academy of Medical Sciences, All Rights Reserved. 24/80
(59). Rubella is also known as

a. The brown measles

b. The German measles

c. The 7-day measles

d. Roseola

Solution. Ans-59: (b) The German measles Ref: Read the text below. Sol:
Rubella is commonly known as the German measles, or the 3-day measles. It has also been called the easy measles, since the symptoms
are quite mild.
After an incubation period of 1421 days, German measles causes symptoms that are similar to the flu.
The primary symptom of rubella virus infection is the appearance of a rash (exanthem) on the face which spreads to the trunk and limbs
and usually fades after three days (that is why it is often referred to as three-day measles). The facial rash usually clears as it spreads to
other parts of the body.
Other symptoms include low grade fever, swollen glands (sub occipital & posterior cervical lymphadenopathy), joint pains, headache and
conjunctivitis.
The swollen glands or lymph nodes can persist for up to a week and the fever rarely rises above 38 oC (100.4 oF).
The rash of German measles is typically pink or light red. The rash causes itching and often lasts for about three days. The rash
disappears after a few days with no staining or peeling of the skin.
When the rash clears up, the patient may notice that his skin sheds in very small flakes wherever the rash covered it. Forchheimer's sign
occurs in 20% of cases, and is characterized by small, red papules on the area of the soft palate.

Your Answer. c
Correct Answer. b

(60). The most common AIDS-defining illness in children less than 13 years old is

a. Epstein Barr infection

b. Gastroenteritis

c. Haemophilus influenzae

d. Lymphoid interstitial pneumonia

Solution. Ans-60: (d) Lymphoid interstitial pneumonia Ref: Read the text below. Sol:
Lymphoid interstitial pneumonia is very common in pediatric HIV + patients.
It is more common than Pneumocystis or any other bacterial or fungal infection.
It is often the first indication of HIV infection.

Your Answer. d
Correct Answer. d

(61). The TIS Classification for Kaposis sarcoma (KS) is based on

a. Lymph node involvement, systemic symptoms, and fever

b. Number of lesions, biopsy characteristics, response to therapy

c. Number of lesions, locations, and visceral involvement.

d. Tumor site(s), CD4+ count, systemic illness

e. Tumor size, CD4+ count, visceral involvement

Solution. Ans-61: (d) Tumor site(s), CD4+ count, systemic illness Ref: Read the text below. Sol:
The National Institute of Allergy and Infectious Diseases AIDS Clinical Trials Group TIS Classification for Kaposis sarcoma consists of
tumor location and size, immune status (CD4+ count), and systemic illness (b symptoms, Karnofsky performance status, history of
opportunistic infection, lymphoma, or thrush).
The classification system is used as an indicator of prognosis and as a guide to choosing therapy.

Your Answer. b
Correct Answer. d

Copyright 2014 Delhi Academy of Medical Sciences, All Rights Reserved. 25/80
(62). Pulmonary KS is commonly mistakenly diagnosed as

a. CMV pneumonia

b. Lymphoma

c. Pneumocystis carinii pneumonia

d. Pneumococcus

Solution. Ans-62: (c) Pneumocystis carinii pneumonia Ref: Read the text below. Sol:
Kaposis sarcoma in the lung is fairly uncommon, but must be kept in mind whenever a patient who is HIV+ presents with dyspnea.
KS commonly causes pleural effusions, but may appear without effusions.
PCP remains the most common respiratory disease in HIV disease; failure to respond to PCP therapy should prompt to search for other
diagnoses, among them KS.

Your Answer. a
Correct Answer. c

(63). Which is the most common bacterial food-borne pathogen in the developed countries?

a. Shigella sonnei

b. Escherichia coli 0157 : H7

c. Salmonella enteritidis

d. Shigella dysenteriae

Solution. Ans-63: (c) Salmonella enteritisdis Ref: Read the text below. Sol:
Salmonella enteritidis is associated with consumption of raw or undercooked eggs, unpasteurized milk, and poultry products and is
currently the most frequent cause of bacterial food-borne illness.

Your Answer. d
Correct Answer. c

(64). Which of the following organism requires notification of the lab for a special agar for identification?

a. Campylobacter jejuni

b. Salmonella enteritidis

c. Escherichia coli 0157: H7

d. Salmonella Heidelberg

Solution. Ans-64: (c) Escherichia coli 0157: H7 Ref: Read the text below. Sol:
E coli 0157:H7 requires a special agar that is different from routine stool cultures fro enteric pathogens.
The colonies are then assayed for the 0157 antigen.

Your Answer. a
Correct Answer. c

Copyright 2014 Delhi Academy of Medical Sciences, All Rights Reserved. 26/80
(65). The most common symptom of pinworm infection is

a. Diarrhea

b. Abdominal pain

c. Perianal pruritus

d. Flatulence

Solution. Ans-65: (c) Perianal pruritus Ref: Read the text below. Sol:
The adult female migrates from the anus to deposit thousands of fertilized eggs on the perianal skin, resulting in pruritus.

Your Answer. c
Correct Answer. c

(66). True about small pox eradication all except

a. Produce life long immunity

b. Vaccine is very effective

c. Cross protection with animal pox

d. Sub clinical infection did not transmit infection

Solution. Ans-66: (c) Cross protection with animal pox . Ref: Parks 20th edition Pg 131.. Sol: It is useful to consider the epidermiological
factors which have led to the eradication of small pox
No known animal reservoir
No long term carrier of the virus
Life long immunity after recovery from the disease
The detection of cases comparatively simple because the rashwas so characteristic and occurred in visible parts of the body
Persons with subclinical infection did not transmit the disease
Vaccine is highly effective; easily administered, heat stable and confers life long immunity
International cooperation
After the eradication of small pox two potential source of infection of human still remain
Accidental infection with laboratory associated stock
Infection with animal pox virus

Your Answer. c
Correct Answer. c

Copyright 2014 Delhi Academy of Medical Sciences, All Rights Reserved. 27/80
(67). Regarding varicella false is

a. Produce rash in flexor surface of fore arm

b. Single stage lesion are found at a time

c. Secondary infection rate is >90%

d. Vaccine protection rate is high

Solution. Ans-67: (b) Single stage lesion are found at a time. Ref: Jawetz, Melnick, & Adelberg's Medical Microbiology, 24th ed pg 248,
Park 20th ed pg 133 Sol:
Varicella (chickenpox) is a mild, highly contagious disease, chiefly of children, characterized clinically by a generalized vesicular
eruption of the skin and mucous membranes.
The disease may be severe in adults and in immunocompromised children.
Zoster (shingles) is a sporadic, incapacitating disease of adults or immunocompromised individuals that is characterized by a rash
limited in distribution to the skin innervated by a single sensory ganglion.
The lesions are similar to those of varicella.
Varicella is the acute disease that follows primary contact with the virus, whereas zoster is the response of the partially immune host to
reactivation of varicella virus present in latent form in neurons in sensory ganglia.
Subclinical varicella is unusual. The incubation period of typical disease is 1021 days.
Malaise and fever are the earliest symptoms, soon followed by the rash, first on the trunk and then on the face, the limbs, and the buccal
and pharyngeal mucosa in the mouth.
Successive fresh vesicles appear in crops, so that all stages of macules, papules, vesicles, and crusts may be seen at one time.
The rash lasts about 5 days, and most children develop several hundred skin lesions.
Park writes
Chicken pox is highly contagious disease and secondary attack rate in house hold contacts is >90%
Live attenuated varicella virus vaccine is safe and currently recommended for children between 12-18 months of age
It has been suggested that it is 90% effective in preventing varicella in an out break, particularly when given within 3-5 days of exposure

Your Answer. c
Correct Answer. b

(68). At what level CD4 + T helper cell count do the majority of HIV infected patients first start to exhibit manifestations of opportunistic
infection ?

< 50>a.

< 100>b.

< 200>c.

< 500>d.

Solution. Ans-68: (c) < 200 Ref: Read the text below Sol :
Most opportunistic infections, including Pneumocystis carinii pneumonia, the most common
AIDS-defining illness in adults, manifest with a CD4+ count below 200.
Many of the less common infections, including Mycobacterium avium intracellulare, are manifest at lower CD4+ cell counts.

Your Answer. c
Correct Answer. c

Copyright 2014 Delhi Academy of Medical Sciences, All Rights Reserved. 28/80
(69). Which of the following organism requires notification of the lab for a special agar for identification ?

a. Campylobacter jejuni

b. Salmonella enteritidis

c. Escherichia coli O157:H7

d. Salmonella Heidelberg

Solution. Ans-69: (c) Escherichia coli O157:H7 Ref: Read the text below Sol :
E. coli 0157: H7 requires a special agar that is different from routine stool cultures for enteric pathogens.
The colonies are then assayed for the 0157 antigen

Your Answer. c
Correct Answer. c

(70). Hepatitis C is associated with all except

a. Porphyria cutanea tarda

b. PAN

c. Essential cryoglobulinaemia

d. Lichen planus

Solution. Ans-70: (b) PAN Ref: Harrisons- 319 Sol :


In porphyria cutanea tarda a number of susceptibility factors, in addition to inherited UROD mutations in type 2 PCT, can be recognized
clinically and can affect management.
These include hepatitis C, HIV, excess alcohol, elevated iron levels, and estrogens.
They syndrome of essential mixed cryoglobulinaemia is strongly associated with hepatitis C virus infection; hepatitis C virions and
hepatitis C virus antigen-antibody complexes have been identified in the cryoprecipitate of these patients.

Your Answer. b
Correct Answer. b

Copyright 2014 Delhi Academy of Medical Sciences, All Rights Reserved. 29/80
(71). All are true regarding disinfectants except

a. Glutaraldehyde is sporicidal

b. Hypochlorites are virucidal

c. Ethylene oxide is intermediate disinfectant

d. Phenol usually requires organic matter to act

Solution. Ans 71: (c) Ethylene oxide is intermediate disinfectant Ref Read the text below Sol Sterilisation refers to a physical or chemical
process that completely destroys or removes all forms of viable microorganisms from an object, including spores. Sterility is an absolute
condition - an item is either sterile or not sterile - there is no such thing as 'partially sterile'. Disinfection describes a process that eliminates
many or all pathogenic microorganisms on inanimate objects, with the exception of bacterial spores.Disinfection can be accomplished by a
number of means that include heat and chemicals. Terminology, which
has been adopted by the Center for Disease
Control (CDC) and is now widely used, describes disinfectants in terms of their activity as set out below:
High-level disinfectants are chemical sterilants, which when used for a shorter exposure period than would be
required for sterilisation, kill all microorganisms with the exception of high numbers of bacterial spores.
Intermediate-level disinfectants may kill mycobacteria, vegetative bacteria, most viruses, and most fungi but do
not necessarily kill bacterial spores
Low-level disinfectants may kill most vegetative bacteria, some fungi, and some viruses.
In 1968 Earle Spaulding devised a rational approach to disinfection and sterilisation. This is now referred to as
Spaulding's classification and it has been refined and retained over the years because it is so clear and logical.
Spaulding believed that instruments and equipment should be cleaned and reprocessed according to the level of
risk associated with their intended use. The three categories he described were critical, semicritical and noncritical
as in the table

Your Answer. b
Correct Answer. c

Copyright 2014 Delhi Academy of Medical Sciences, All Rights Reserved. 30/80
(72). Acellular pertusis vaccine contains

a. Pertactin, flagillary hemagglutinin, cytotoxin, endotoxin

b. Pertactin, flagillary hemagglutinin, fimbriae, endotoxin

c. Pertactin, cytotoxin, fimbriae, pertactin,

d. Flagillary hemagglutinin, pertusis toxin,fimbriae

Solution. Ans 72: (d) Flagillary hemagglutinin, pertusis toxin,fimbriae


Ref Read the text below
Sol
Acellular pertussis vaccines contain inactivated pertussis toxin (PT) and may contain one or more other
bacterial components (e.g., filamentous hemagglutinin [FHA], a 69-kilodalton outer-membrane proteinpertactin
[Pn], and fimbriae [Fim] types 2 and 3).PT is detoxified either by treatment with a chemical (e.g., hydrogen peroxide, formalin and/or
glutaraldehyde) or by using molecular genetic techniques. Acellular pertussis vaccines contain substantially less
endotoxin than whole-cell pertussis vaccines.

Your Answer. d
Correct Answer. d

(73). A large majority of fatal transfusion reactions result from:

a. Anti-IgA antibodies

b. Antibodies to HLA antigens

c. Bacterial contamination of transfused blood

d. Human error

Solution. Ans-73: (d) Human error Ref: Read the text below Sol:
Human error is the most frequent cause, and bacterial contamination of transfused blood is the second most frequent cause of fatal
transfusion reactions.

Your Answer. d
Correct Answer. d

(74). Effective mode of sterilization is :

a. Hot water

b. Steam under pressure

c. Steam at atmospheric pressure

d. Dry heat

Solution. Ans-74: (b) Steam under pressure Ref: Ananthanarayans - 33 Sol :


Boiling and steam at atmospheric pressure are ineffective means of killing bacterial spores.
Boiling requires prolonged period of boiling for killing spores whereas steam at atmospheric pressure may fail with spores of certain
anaerobes and thermophiles. Nothing short of autoclaving at thigh pressure can destroy spores and ensure sterilization.

Your Answer. d
Correct Answer. b

Copyright 2014 Delhi Academy of Medical Sciences, All Rights Reserved. 31/80
(75). Cryptococcus is transmitted to humans via?

a. Aerosolized particles from soil

b. Animal bitesc. Contaminated water

c. Contaminated water

d. Human-to-human contact

Solution. Ans-75: (a) Aerosolized particles from soil Ref: Read the text below Sol :
Cryptococcus is endemic in pigeons and other birds and excreted in their droppings.
The fungus remains in the soil and can be inhaled in aerosolized particles when the soil is disturbed.
There is no evidence of human-to-human or animal (insect)-to-human transmission.

Your Answer. a
Correct Answer. a

(76). Funduscopic changes typical of CMV retinitis include

a. Copper spots

b. Hyperpigmentation

c. Neovasularization

d. White, cheesy exudates

Solution. Ans-76: (d) White, cheesy exudates Ref: Read the text below Sol :
White to yellow cheese-like exudates are they typical characteristics of CMV retinitis.

Your Answer. b
Correct Answer. d

(77). True about autoclaving is :

a. 115 degree C at for 20 min

b. 121 degree C at for 15 min

c. 118 degree C at for 15 min

d. 124 degree C at for 15 min

Solution. Ans-77: (b) 121 degree C at for 15 min Ref: Ananthanarayans -.32 Sol : Heat sterilization.
A widely-used method for heat sterilization is the autoclave.
Autoclaves commonly use steam heated to 121 C or 134 C.
To achieve sterility, a holding time of at least 15 minutes at 121 C or 3 minutes at 134 C required.
Additional sterilizing time is usually required for liquids and instruments packed in layers of cloth, as they may take longer to reach the
required temperature.
After sterilization, autoclaved liquids must be cooled slowly to avoid boiling over when the pressure is released.

Your Answer. b
Correct Answer. b

Copyright 2014 Delhi Academy of Medical Sciences, All Rights Reserved. 32/80
(78). Shigella are be divided into sub group on the basis of ability to ferment:

a. Lactose

b. Maltose

c. Fructose

d. Mannitol

Solution. Ans-78: (d) Mannitol Ref.: Read the text below Sol :
All Shigella are mannitol fermenter except Sh. Dystenteriae.

Your Answer. d
Correct Answer. d

(79). Which antigen blocks the agglutination of salmonella by 0 antiserum:

a. H

b. Fimbriae

c. Vi

d. 0

Solution. Ans-79: (c) Vi Ref.: Read the text below Sol :


Vi antigen Polysaccharide antigen enveloping the O antigen because of which many strains of S. typhi fails to agglutinate with the O
antiserum.Antigens of Salmonella :

Your Answer. a
Correct Answer. c

(80). Maximum urease is produced by :

a. H. pylori

b. P. Mirabilis

c. K. rhinomatis

d. Ureaplasma

Solution. Ans-80: (b) P. Mirabilis Ref.: Read the text below Sol : Urease production (in order of decreasing) :
Proteus
Klebsiella
Citrobacter

Your Answer. a
Correct Answer. b

Copyright 2014 Delhi Academy of Medical Sciences, All Rights Reserved. 33/80
(81). Which of the following factor is mainly responsible for virulence in streptococcus :

a. Carbohydrate

b. Streptokinase

c. Streptodornase

d. M-Protein

Solution. Ans-81: (d) M-Protein Ref.: Read the text below Sol : M Protein
It is major virulence factor of group A strep. Pyogenes.
It is hair-like projection of cell wall.
It is of 80 types (basis of Griffith classification) so immunity is type specific.
M protein is also found in Group G streptococci.

Your Answer. d
Correct Answer. d

(82). A 7% sodium dodecyl sulfate polyacrylamide gel electrophoretogram of E. coli cell walls is shown below. The gel is labeled AE.

The molecular weight of lactose permease is represented by

a. A

b. B

c. C

d. D

Solution. Ans 82: (d) D Ref :Read the text below Sol:
Gel electrophoresis provides a rapid method for identifying bacterial proteins and estimating molecular weights.
A gel can be made of a number of substances, including starch, agar, and polyacrylamide.
Starch gel has high separating power because the fine gel pores act as a molecular sieve. Agar gel is easier to prepare than starch;
separation of proteins is accomplished in 30 to 60 min.
Polyacrylamide gel also separates on the principle of the molecular sieve. It is chemically inert and electrically neutral.
The biggest disadvantage of polyacrylamide is that its separating powers are so good that protein patterns, or patterns of other
heterogeneous substances, may be too complex to interpret.
In the electrophoretogram presented in the question, band A represents RNA polymerases (molecular weight 155,000), band C
represents both flagellin and the major cell-wall protein (50,000), and band D represents lactose permease (30,000). Band E is the dye
front.

Your Answer. c
Correct Answer. d

(83). Which of the following is false about mycetoma :

a. Can affect lower and upper extremities

b. Caused by actinomycetes and filamentous fungi

c. Diagnosis is by examination of pus

d. Uncommon in India.

Solution. Ans-83: (d) Uncommon in India. Ref.: Read the text below Sol :
Most cases are reported from dry tropics because of the habit of walking barefoot.
It was first decribed from Madural (in South India) so it is quite common in India.

Your Answer. d
Correct Answer. d

Copyright 2014 Delhi Academy of Medical Sciences, All Rights Reserved. 34/80
(84). Lance field grouping of streptococci is done by using :

a. M protein

b. Group C peptidoglycan cell wall

c. Group C carbohydrate antigen

d. M antigen

Solution. Ans-84: (c) Group C carbohydrate antigen Ref.: Read the text below Sol : Lancefield classification
Classification of hemolytic streptococci into Group A to V (except I, J) on the basis of group specific C carbohydrate.
Griffith classification
Serological typing of group A streptococcus pyogenes on the basis of M proteins into types 1, 2, 3 etc.
Group C Carbohydrate
Present in middle layer or cell wall.
This antigen is an integral part of cell wall

Your Answer. c
Correct Answer. c

(85). Pus with brownish granules is seen in :

a. Nocardia asteroids

b. Madurella mycetoma

c. Actinomadura pelletierii

d. Alleschirea boydii

Solution. Ans-85: (c) Actinomadura pelletierii Ref.: Read the text below Sol :
Mycetoma may be caused by Actinomycetes, which are considered as transitional forms between bacteria and fungi or by true fungi
(Eumycetes) where it is called Eumycetoma.Bacterial and true fungal species that can cause mycetoma are listed below under their
characteristic colours of discharge from infected wounds: Red discharge
Actinomadura pelletieri
White or Yellow discharge
Acremonium strictum
Actinomadura madurae
Aspergillus nidulans
Noetestudina rosatii
Phaeoacremonium krajdenii[6]
Pseudallescheria boydii[7]
Black discharge
Aspergillus terreus
Curvularia lunata
Cladophialophora bantiana
Exophiala jeanselmei[8]
Leptosphaeria senegalensis
Leptosphaeria tompkinsii
Madurella grisea[9]
Madurella mycetomatis[10]
Pyrenochaeta romeroi

Your Answer. a
Correct Answer. c

Copyright 2014 Delhi Academy of Medical Sciences, All Rights Reserved. 35/80
(86). All of the following statement are true about staphylococci except:

a. A majority of infection caused by coagulase (-) ve staphylococci are due to staph. epidemidis

b. b-Lactamase production is under plasmid control

c. Expression of methicillin resistance in Staphylococcus aureus increases when it is incubated at 37C on blood agar

d. Methicillin resistance in Staph. aureus is independent of b-Lactamase production

Solution. Ans-86: (c) Expression of methicillin resistance in Staphylococcus aureus increases when it is incubated at 37C on blood agar
Ref.: Read the text below Sol :
Methicillin resistance is expressed more when staphylococci is incubated at 30C than at 37C.
Penicillin resistance in staphylococcus is by following ways:
Production of lactamase.
Plasmid mediated inducible enzyme which is transmitted by transduction (more commonly) or conjugation.
Now only <5% of strains of staph are sensitive to penicillin.
Hospital strains mostly form type A penicillinase.
Same plasmid carry genes for resistance to tetracycline, erythromycin, aminoglycoside too.

Your Answer. a
Correct Answer. c

(87). Patients with severe neutropenia and fever are particularly susceptible to which one of the following infections :

a. Pneumocystis carinii

b. Toxoplasma gondii

c. Cytomegalovirus

d. Candidiasis

Solution. Ans-87: (d) Candidiasis Ref: Davidsons Principles and Practice of Medicine--756 Sol:
Neutropenic patients are predisposed to invasive fungal infections most commonly due to Candia, Aspergillus and occasionally due to
Fusarium, Trichosporon, Bipolaris.
Example of potential pathogens in neutopenic patient.

Your Answer. a
Correct Answer. d

Copyright 2014 Delhi Academy of Medical Sciences, All Rights Reserved. 36/80
(88). Cold sterilization is done by :

a. Steam

b. Inionizing radiation

c. Infra red

d. UV

Solution. Ans-88: (b) Inionizing radiation Ref.: Ananthanarayans Microbiology, 8th ed, p-35 Sol :
Cold sterilization is a process in which sterilization is carry out at low temperature with the help of chemicals, radiations, membranes
and all other means excluding high temperature.
Ionizing radiation such as X-rays, gamma rays and cosmic rays, cause no appreciable increase in temperature in this method, it is
referred as cold sterilization.

Your Answer. c
Correct Answer. b

(89). Intravascular hemolysis is mediated by :

a. IgA

b. IgD

c. IgE

d. IgG

Solution. Ans-89: (d) IgG Ref.: Ananthanarayans Microbiology, 8th ed, p-166 Sol :
In a transfusion reaction or in alloimmune hemolytic anemia antibody mediated lysis of red blood cells involves triggering of the
complement cascade.
Some microbes form substances called hemolysins that have the specific action of destroying red blood cells; beta hemolytic
streptococci are an example.
Intravenous administration of a hypotonic solution or plain distilled water will cause te red cells to fill with fluid until their membranes
rupture and the cells are destroyed.
Wherever either in vitro or in vivo IgG or IgM antibodies are bound to red blood cell antigens in the presence of complement, the
complement cascade is triggered the final products of which include enzymes that result in holes being punched in the wall of the red
blood cell, allowing hemoglobin to escape and which is observed as lysis.

Your Answer. d
Correct Answer. d

(90). A patient with stage 2 Lyme disease will most likely present with which of the following problems ?

a. Bells palsy

b. Pneumonitis

c. Glomerulonepritis

d. Erythema nodosum

Solution. Ans- 90: (a) Bells palsy Ref.: Read the text below Sol :
Cranial neuropathies, particularly facial neuropathies, are quite common in stage 2 of Lyme disease.
The neuropathies may be painful and can last up to 6 to 8 months.
Peripheral neuropathies are manifested with intermittent burning paresthesias and weakness

Your Answer. c
Correct Answer. a

Copyright 2014 Delhi Academy of Medical Sciences, All Rights Reserved. 37/80
(91). A patient presents with lower gastrointestinal bleed. Sigmoidoscopy shows ulcers in the sigmoid. Biopsy from this area shows flask-shaped
ulcers. Which of the following is the most appropriate treatment?

a. Intravenous ceftriaxone

b. Intravenous metronidazole

c. Intravenous steroids and sulphasalazine

d. Hydrocortisone enemas

Solution. Ans-91: (b) Intravenous metronidazole Ref.: Read the text below Sol :
It is case of intestinal amoebiasis in the form of amoebic dysentery
Drug Therapy for Amebiasis

Your Answer. b
Correct Answer. b

(92). The following five growth curves are lettered (AE) corresponding to an expected growth curve if certain antibiotics were added to an
exponentially growing culture of E. coli. The arrow indicates when antibiotics were added to the growing culture.

Chloramphenicol treatment would be expected to produce which one of the following growth curves?

a. A

b. B

c. C

d. D

Solution. Ans 92: (d) D Ref :Read the text below Sol:
Penicillin causes lysis of growing bacterial cells. Its antimicrobial effect stems from impairment of cell-wall synthesis.
Because penicillin is bactericidal, the number of viable cells should fall immediately after introduction of the drug into the medium.
Both chloramphenicol and sulfonamides are bacteriostaticthat is, they retard cell growth without causing cell death.
Chloramphenicol causes an immediate, reversible, bacteriostatic inhibition of protein synthesis.
Sulfonamides, on the other hand, compete with para-aminobenzoic acid in the synthesis of folate; intracellular stores of folate are
depleted gradually as the cells continue to grow.

Your Answer. c
Correct Answer. d

Copyright 2014 Delhi Academy of Medical Sciences, All Rights Reserved. 38/80
(93). Which of the following organisms has diverse animal reservoirs?

a. Vibrio cholerae

b. Vibrio parahaemolyticus

c. Salmonella typhi

d. Shigella sonnei

Solution. Ans-93: (c) Salmonella typhi Ref: Read the text below Sol :
Salmonella is associated with intestinal tracts of humans, other animals, reptiles, amphibians, fish, and birds.

Your Answer. b
Correct Answer. c

(94). Which of the following organisms causes acute icteric disease with protean manifestations?

a. Treponema

b. Borrelia

c. Leptospira

d. None

Solution. Ans-94: (c) Leptospira Ref: Read the text below Sol :
The acute icteric disease caused by Leptospira is characterized by abrupt onset of fever, chills, headache, myalgia, gastrointestinal
upset, conjunctival suffusion, and aseptic meningitis.

Your Answer. c
Correct Answer. c

(95). Which statement about the two major forms of leprosy (tuberculoid and lepromatous) is correct?

a. Tuberculoid leprosy is the hardest to treat due to poor cell-mediated immunity.

b. In either extreme form of leprosy, the lepromin test will be positive.

c. Diagnostic work-up on the patient should include a physical examination, skin test, and cultures.

d. Biopsy sample of lesions from a lepromatous leprosy patient generally contains many Mycobacterium leprae.

Solution. Ans-95: (d) Biopsy sample of lesions from a lepromatous leprosy patient generally contains many Mycobacterium leprae. Ref:
Read the text below Sol :
Numerous Mycobacterium leprae organisms are found in the skin in lepromatous leprosy, and the patient has many skin lesions from
failure of cell-mediated immunity to restrict infection.
Lepromatous leprosy patients usually have a poor response to the lepromin skin test.
Leprosy is diagnosed by cytologic findings and a lepromin skin test, not by culture, because the organism cannot be cultured in vitro.

Your Answer. c
Correct Answer. d

Copyright 2014 Delhi Academy of Medical Sciences, All Rights Reserved. 39/80
(96). A patient presents with paranasal swelling and bloody exudate from both his eyes and nares, and he is nearly comatose. What is the most
likely compromising condition underlying this infection caused by Rhizopus, Mucor, or Absidia (phylum Zygomycota, class
Phycomycetes)?

a. AIDS

b. Diabetes (with patient in ketoacidosis)

c. Neutropenia

d. B-cell defects

Solution. Ans-96: (b) Diabetes (with patient in ketoacidosis) Ref: Read the text below Sol :
Zygomycota are aseptate fungi that cause serious infections, primarily in ketoacidotic diabetic patients and cancer patients.
Fungal infections common in AIDS patients include Candida infections (ranging from oral thrush early to fungemias later), cryptococcal
meningitis, and disseminated histoplasmosis and coccidioidomycoses.
Severely neutropenic patients are most likely to have invasive Aspergillus infections.

Your Answer. c
Correct Answer. b

(97). What type of fungal skin lesions would show only hyphae and possible arthroconidia in a biopsy?

a. Blastomycosis (disseminated)

b. Chronic mucocutaneous candidiasis

c. Coccidioidomycosis

d. Dermatophytosis (e.g., tinea pedis)

Solution. Ans-97: (d) Dermatophytosis (e.g., tinea pedis) Ref: Read the text below Sol :
Of the choices listed, only the dermatophytes would show hyphae and arthroconidia and cause cutaneous lesions. Blastomycosis (caused
by Blastomyces) would show big, broad-based, budding yeasts.Candidiasis would show yeasts, pseudohyphae, and true hyphae.
Coccidioidomycosis would show spherules of various sizes with round endospores visible in the large, mature spherules.
Pityriasis versicolor would have clusters of yeasts with short, septate, curved hyphae (spaghetti and meatball appearance).

Your Answer. d
Correct Answer. d

(98). Which of the following immunoglobulins (Ig) is at its highest level in a normal adult?

a. IgA

b. IgG

c. IgM

d. IgD

Solution. Ans-98: (b) IgG Ref: Read the text below Sol :
Immunoglobulin G (IgG) is the predominant immunoglobulin (approximately 73% of total immunoglobulin) found in the adult.

Your Answer. b
Correct Answer. b

Copyright 2014 Delhi Academy of Medical Sciences, All Rights Reserved. 40/80
(99). Which of the following autoimmune diseases is associated with defective neutrophil bactericidal activity resulting from depressed
superoxide dismutase?

a. Ulcerative colitis

b. Multiple sclerosis

c. Chronic granulomatous disease

d. Congenital thymic aplasia

Solution. Ans-99: (c) Chronic granulomatous disease Ref: Read the text below Sol :
Chronic granulomatous disease is characterized by a genetic defect in the nicotinamide adenine dinucleotide phosphate (NADPH)
oxidase system. It may be treated with interferon.

Your Answer. c
Correct Answer. c

(100). Scotochromogens are :

a. Mycobacterium gordonae

b. Mycobacterium marinum

c. Mycobacterium intracellulare

d. Mycobacterium avium

Solution. Ans-100: (a) Mycobacterium gordonae Ref: Greenwood 16/e, p 216 Sol : Scotochromogens includes :
M. szulgai
M. scrofulaceum
M. gordonae/M. acquae

Your Answer. b
Correct Answer. a

(101). A 20-year-old male had pain in abdomen and mild fever followed by gastroenteritis. The stool examination showed presence of pus cells
and RBCs on microscopy. The most likely etiological agent is :

a. Enteroinvasive E. coli

b. Enteropathogenic E. Coli

c. Enterotoxigenic E. Coli

d. Enteroaggregative E. Coli

Solution. Ans-101: (a) Enteroinvasive E. coli Ref: Ananthnarayan -277 Sol :


EIEC cause illness resembling Shigellosis i.e. ranging from mild diarrhea to frank dysentery.

Your Answer. a
Correct Answer. a

Copyright 2014 Delhi Academy of Medical Sciences, All Rights Reserved. 41/80
(102). For typhoid endemic country like India, immunization of choice is :

a. TAB vaccine

b. Typhoral 21A oral vaccine

c. Monovalent vaccine

d. Any of these

Solution. Ans-102: (c) Monovalent vaccine Ref: Park - 197 Sol :


Since S. typhi is the major cause of typhoid fever in india, the vaccine of choice is the monovalent typhoid vaccine.
Antityphoid vaccine in India are :
o Monovalent antityphoid vaccine Heat killed and phenol preserved.
o Bivalent antityphoid vaccine Contains S. typhi and paratyphi A.
o TAB vaccine (WHO recommended that TAB vaccine should be discontinued).
Live oral typhoid 21a vaccine (Typhoral)
Enteric coated capsule of lyophilized vaccine containing not less than 10 viable organism of attenuated S. typhi strain Ty 21a.
It is indicated for immunization of adults and children aged more than 6 years.
Protection commences 2 weeks after taking last capsule and last for at least 3 years.
Dose 1 capsule on days 1, 3 and 5 one hour before meal with cold or luke warm milk or water.

Your Answer. b
Correct Answer. c

(103). Disinfectant used as a standard of measurement of killing power.

a. Alcohol

b. Phenol

c. Cresol

d. Formaldehyde

Solution. Ans-103: (b) Phenol Ref.: Read the text below Sol :
Chemical disinfectants vary greatly in their ability to kill microorganisms.
A quantitative measure of this variation is expressed as the phenol coefficient, which is the ratio of the concentration of phenol to the
concentration of the agent required to cause the same amount of killing under the standard conditions of the test.

Your Answer. a
Correct Answer. b

Copyright 2014 Delhi Academy of Medical Sciences, All Rights Reserved. 42/80
(104). Which of the following is most resistant to antiseptics?

a. Spore

b. Prior

c. Cyst

d. Fungus

Solution. Ans-104: (b) Prior Ref.: Read the text below Sol : Resistance of organism to antiseptics in decreasing order is as follows :
Prions
Coccidia
Spores
Mycobacteria
Cysts
Small non-enveloped virus
Trophozoites
Gram-negative bacteria
Fungi
Large non-enveloped virus
Gram-positive bacteria
Lipid enveloped/medium size virus (HIV, HBV)

Your Answer. b
Correct Answer. b

(105). Which of the following statement is true:

a. Solid media are enrichment media

b. Nutrient broth is basal media

c. Agar adds nutrient to media

d. Chocolate agar is selective media.

Solution. Ans-105: (b) Nutrient broth is basal media Ref.: Read the text below Sol :

Your Answer. d
Correct Answer. b

Copyright 2014 Delhi Academy of Medical Sciences, All Rights Reserved. 43/80
(106). Rabies virus persists in the saliva for how many days:

a. 3-6

b. 7-10

c. 1-3

d. Upto 6 years

Solution. Ans-106: (a) 3-6 Ref.: Read the text below Sol : SOURCE OF RABIES INFECTION
The source of infection to man is the saliva of rabid animals.
In dogs and cats, the virus may be present in the saliva for 3-4 days (occasionally 5-6 days) before the onset of clinical symptoms and
during the course of illness till death.
However, it may be stated that not all rabid animals have virus in saliva and even if present, the quantity is variable.
The variability of the virus in saliva explains the fact why only about 50% of bites by proven rabid animals will result in rabies.

Your Answer. b
Correct Answer. a

(107). Tubercle bacilli was discovered by:

a. Robert Koch

b. Hansen

c. Edward Jenner

d. Virchow

Solution. Ans-107: (a) Robert Koch Ref.: Read the text below Sol :
On March 24, 1882, Robert Koch announced to the Berlin Physiological Society that he had discovered the cause of tuberculosis.
Three weeks later, on April 10, he published an article entitled "The Etiology of Tuberculosis"
In 1884, in a second paper with the same title, he first expounded "Koch's postulates," which have since become basic to studies of all
infectious diseases.
He had observed the bacillus in association with all cases of the disease, had grown the organism outside the body of the host, and had
reproduced the disease in a susceptible host inoculated with a pure culture of the isolated organism.

Your Answer. a
Correct Answer. a

Copyright 2014 Delhi Academy of Medical Sciences, All Rights Reserved. 44/80
(108). A 6 year old boy presents with lesions on dorsal aspect of hands and genitals as shown in image.What is the diagnosis?

a. Molluscum contagiosum

b. Scabies

c. Lichen nitidus

d. Lichen planus

Solution. Ans-108: (c) Lichen nitidus Ref.: Read the text below Sol :
Lichen nitidus presents as multiple grouped pinhead sized shiny papules mainly in children. Cause unknown. Sites-genitals, arms.
Biopsy has a claw clutching a ball appearance. Usually self limiting.
Topical steroids and tacrolimus may be used.

Your Answer. a
Correct Answer. c

(109). Pneumococcus producing mucoid colonies most often is type:

a. I

b. II

c. III

d. IV

Solution. Ans 109: (c) III Reference Read the text below Sol:
Pneumococci are identified in the clinical laboratory as catalase-negative, gram-positive cocci that grow in pairs or chains and cause -
hemolysis on blood agar.
More than 98% of pneumococcal isolates are susceptible to ethylhydrocupreine (optochin), and virtually all pneumococcal colonies are
dissolved by bile salts.
Peptidoglycan and teichoic acid are the principal constituents of the pneumococcal cell wall, whose integrity depends on the presence of
numerous peptide side chains cross-linked by the activity of enzymes such as trans- and carboxypeptidases.
Some strains that develop abundant capsular material (type 3 and 7) from large mucoid colonies.

Your Answer. b
Correct Answer. c

Copyright 2014 Delhi Academy of Medical Sciences, All Rights Reserved. 45/80
(110). A man with chills, fever, and headache is thought to have "atypical" pneumonia. History reveals that he raises chickens and that
approximately 2 weeks ago he lost a large number of them to an undiagnosed disease. Which of the following is the most likely diagnosis
of this man's condition?

a. Q fever

b. Relapsing fever

c. Leptospirosis

d. Ornithosis (psittacosis)

Solution. Ans 110: (d) Ornithosis (psittacosis) Reference Read the text below Sol: Ornithosis (psittacosis) is caused by Chlamydia
psittaci.
Humans usually contract the disease from infected birds kept as pets or from infected poultry, including poultry in dressing plants.
Although ornithosis may be asymptomatic in humans, severe pneumonia can develop. Fortunately, the disease is cured easily with
tetracycline

Your Answer. d
Correct Answer. d

(111). Which of the following mycoplasmas has been implicated as a cause of nongonococcal urethritis (NGU)?

a. M. pneumoniae

b. M. fermentans

c. M. mycoides

d. Ureaplasma urealyticum

Solution. Ans 111: (d) Ureaplasma urealyticum Reference Read the text below Sol:
Ureaplasma urealyticum has been associated with nongonococcal urethritis (NGU) as well as infertility.
Mycoplasma pneumoniae is the etiologic agent of primary atypical pneumonia.
M. hominis, although isolated from up to 30% of patients with NGU, has yet to be implicated as a cause of that disease.
M. fermentans has on rare occasions been isolated from the oropharynx and genital tract. M. mycoides causes bovine pleuropneumonia.

Your Answer. d
Correct Answer. d

(112). Mycobacteriu simiae belongs to which of the following.

a. Scotochromogen

b. Photo chromogen

c. Non photochromogen

d. Rapid grower

Solution. Ans-112: (b) Photo chromogen Ref.: Ananthanarayan-6th Ed. Sol :


Mycobacterium simiae is a photochromogenic mycobacterium that was first isolated from feral monkeys imported into the Unites States.
Mycoacterium simiae shares many similarities with M. avium and M. scrofulaceum and has been associated with infections in HIV
infected individuals, which resemble M. avium-intracellulare disease.
Treatment for significant infections should be with a regimen suitable for M. avium.

Your Answer. b
Correct Answer. b

Copyright 2014 Delhi Academy of Medical Sciences, All Rights Reserved. 46/80
(113). Periportal fibrosis is caused by :

a. Schiztosomia hemotobium

b. Schiztosomia japanicum

c. Schiztosomia mansoni

d. Schiztosomia intercalatum

Solution. Ans-113: (c) Schiztosomia mansoni Ref.: Ananthanarayan-6th Ed. Sol :


Schistosoma mansoni worm produce approximately 300 eggs per day.
Approximately 50% of the deposited eggs are swept into the circulation and filter out in the periportal tracts of the liver eliciting
granulomatous inflammatory reactions that can lead to periportal fibrosis, portal hypertension and the serious sequelae of intestinal
schistosomiasis such as hepatosplenomegaly and esophageal and gastric varicies.
Periportal fibrosis in human Schistosoma mansoni infection is associated with high TNF alpha, low IL 10, low IFN gamma, or low
RANTES.
Egg-induced Th2 cytokine responses such as IL 4, IL -13 and TNF alpha promote hepatic fibrosis, and this is counter regulated by
IL 12 and IFN gamma.

Your Answer. b
Correct Answer. c

(114). Miyagawa granulocorpusles bodies are seen in :

a. Lymphogranuloma venereum (LGV)

b. Chylamydia pneumonia

c. Mycoplasma

d. Chlamydia trachomatosis

Solution. Ans-114: (a) Lymphogranuloma venereum (LGV) Ref.: Ananthanarayan- 6th Ed. Sol : Some important structures to be
remembered are :
Miyagawas granulocorpuscles LGV
Donovan bodies Granuloma lnguinale
Halberstaedter Prowazek Bodies Trachoma
Babes Ernst
Volutin granules : C. diphteriae
Levinthal Cole Lillie bodies : Psittacosis.
Citron bodies Clostridium septicum
Mooser bodies Endemic typhus

Your Answer. d
Correct Answer. a

Copyright 2014 Delhi Academy of Medical Sciences, All Rights Reserved. 47/80
(115). Guaneri bodies are found in :

a. Small Pox

b. Herpes virus

c. Adenovirus

d. Coxsackie virus

Solution. Ans-115: (a) Small Pox Ref.: Ananthanarayan- 6th Ed. Sol :
Inclusion bodies are nuclear or cytoplasmic aggregates of stainable, usually proteins. They typically represent sites of viral
multiplication and usually consist of viral capsid proteins. They can be classified based on location, staining property and size as follows :
Location
Cytoplasm Eg : Poxvirus
Nucleus
Cowdry type A
Variable size & granular
Eg.: herpes, Yellow fever (Type A : HAY)
Cowdry type B
Well circumscribed & multiple
Eg.: Adenovirus, Polio (Type B : BAP)
Both Eg: Measles

Your Answer. c
Correct Answer. a

(116). Sclerotic bodies measuring 3-15m in size, multi-septate,chestnut, brown color is characteristic of:

a. Histoplasmosis

b. Rhinosporodiosis

c. phaeohypomycosis

d. Chromoblastomycosis

Solution. Ans 116: (d) Chromoblastomycosis. Reference Read the text below Sol:
Chromoblastomycosis is characterized by slow-growing verrucous plaques or nodules, usually on the lower extremities.
The most common etiologic agents are Fonsecaea pedrosoi, F. compacta, Phialophora verrucosa, Rhinocladiella aquaspersa, and
Cladosporium (Cladophialophora) carrionii.
Small verrucous papules enlarge slowly but remain painless. Lesions seen in late stages may be superficial or raised purplish irregular
plaques; less commonly, they may be nodular, tumorous, verrucous, or cicatricial. In advanced cases, secondary lymphedema, bacterial
infections, and keratin necrosis can develop.
Although histologic examination of scrapings or biopsy material for characteristic sclerotic bodies can lead to the diagnosis of
chromoblastomycosis, culture is required for identification of the causative agent.
Treatment is difficult, although many therapeutic interventions have been described . Results are best when early surgical excision or
cryosurgery is used in combination with antifungal therapy.
Treatment with itraconazoleeither alone or with 5-fluorocytosinehas had some success.

Your Answer. a
Correct Answer. d

Copyright 2014 Delhi Academy of Medical Sciences, All Rights Reserved. 48/80
(117). Infection with hepatitis D virus (HDV; delta agent) can occur simultaneously with infection with hepatitis B virus (HBV) or in a carrier of
hepatitis B virus because HDV is a defective virus that requires HBV for its replicative function. What serologic test can be used to
determine whether a patient with HDV is an HBV carrier?

a. HBsAg

b. HBc IgM

c. HBeAg

d. HBs IgM

Solution. Ans 117: (b) HBc IgM Reference Read the text below Sol:
In a chronic HBV carrier, there would be no HB core IgM antibody, whereas it would be present in a new HBV infection.
The HBe antigen could be present in either an HBV carrier or in acute infection.
HBsAg would be present in either a new infection or in the carrier state, while HBsAb would not be present in either case.

Your Answer. c
Correct Answer. b

(118). Battery bacillus or mycobacterium intracellular is :

a. Photochromogen

b. Scotochromogen

c. Nonchromogen

d. Rapid grower

Solution. Ans 118: (c) Nonchromogen Reference Read the text below Sol: NONCHROMOGEN
Nonchromogen, dont form pigment even on exposure to light e.g. M. intracellular, m. avium, M. xenopi.

Your Answer. a
Correct Answer. c

(119). Eastern equine encephalitis virus is associated with a high fatality rate. Control of the disease could be possible by eradication of which
of the following species?

a. Birds

b. Mosquitoes

c. Fleas

d. Ticks

Solution. Ans 119: (b) Mosquitoes Reference Read the text below Sol:
Eastern equine encephalitis (EEE) is a severe disease usually seen in the summer months when Aedes mosquitoes are prevalent.
Control of EEE is a function of mosquito eradication.
Horses and humans are accidental hosts.

Your Answer. c
Correct Answer. b

Copyright 2014 Delhi Academy of Medical Sciences, All Rights Reserved. 49/80
(120). Which one of the following Gram positive organism is most common cause of UTI among sexually active women:

a. Staphylococcus epidermidis

b. Staphylococcus aureus

c. Staphylococcus saprophyticus

d. Enterococcus

Solution. Ans-120: (c) Staphylococcus saprophyticus Ref.: Read the text below Sol :
S. saprophyticus cause UTI in young women due to its enhanced capacity to adhere to uroepithelial cells.
A 160 KDa hemagglutinin adhesions may contribute to this affinity.
S. Saprophyticus is novobiocin resistant.

Your Answer. c
Correct Answer. c

(121). In autoclaving, which of the following spore used to confirm effectiveness:

a. Bacillus anthraxb. Bacillus stearotherm

b. Bacillus stearothermophilus

c. Clostridia spores

d. Bacillus cerius

Solution. Ans-121: (b) Bacillus stearothermophilus Ref.: Read the text below Sol : STERILIZATION CONTROL :
For determining the efficacy of moist-heat sterilization, spores of Bacillus stearothermophilus are used as the test organism.
The is a thermophilic organism with an optimum growth temperature of 55-60C and its spores require an exposure of 12 minutes at
121C to be killed.
Paper strips impregnated with 106 spores are dried at room temperature and placed in paper envelopes.
These envelopes are inserted in different parts of a load and after being sterilized; the strips are inoculated into a suitable recovering
medium and incubated for sterility test at 55C for five days.
Chemical indicators, autoclave tapes and thermocouples are also used instead.

Your Answer. c
Correct Answer. b

(122). Which of the following cannot be reliably used for hand washing :

a. Chlorh exidine

b. Isopropyl alcohol

c. Lysol

d. Cresol

Solution. Ans-122: (c) Lysol Ref.: Read the text below Sol : Skin disinfectants are :

Your Answer. d
Correct Answer. c

Copyright 2014 Delhi Academy of Medical Sciences, All Rights Reserved. 50/80
(123). A diabetic patient developed cellulitis due to s.aureus, which was found to be methicillin resistant on the antibiotic sensitivity testing. All
the following antibiotics will be appropriate except:

a. Vancomycin

b. Imipenem

c. Teicloplanin

d. Linezolid

Solution. Ans-123: (b) Imipenem Ref.: Read the text below Sol :
Imipenem is not effective against Enterococcus faecium, MRSA, Clostridium difficle,k Burkholderia cepacia as they produce
metallobetalactamases.
Though it is effective against methicillin sensitive staph. aureus.

Your Answer. b
Correct Answer. b

(124). True regarding tubercle bacilli are all except:

a. Grows at 37C

b. Pigments are produced in colonies

c. Koch first discovered it

d. Slow growing

Solution. Ans-124: (b) Pigments are produced in colonies Ref.: Read the text below Sol :
Koch (1882) isolated the mammalian tubercle bacillus and proved its causative role in tuberculosis by satisfying Kochs postulates.
Tuberculosis in human beings was subsequently shown to be caused by two types of the bacillus the human and bovine types,
designated Mycobacterium tuberculosis and M. bovis, respectively.
Mycobacterium is slender rods that sometimes show branching filamentous forms resembling fungal mycelium. Hence the name
mycobacterium, meaning fungus like bacteria.
They do not stain readily but once stained, resist decolorisation with dilute mineral acids. Mycobacterium is therefore called acid fast
bacilli or AFB,
They are aerobic, non-motile, non-capsulated and non-sporing.
M. tuberculosis does not produce any pigments but many atypical mycobacterium produce pigmented colonies.

Your Answer. b
Correct Answer. b

(125). Plasmodium falciparum malaria is severe because :

a. Hypnozoites in liver

b. Primaquine resistance

c. Infects a larger number of RBCs

d. Non e of the above

Solution. Ans-125: (c) Infects a larger number of RBCs Ref.: Read the text below Sol :
Malaria caused by P falciparum is more severe than that caused by other plasmodia.
It is characterized by infection of far more red cells than the other malarial species and by occlusion of the capillaries with aggregates of
parasitized red cells.
This leads to life-threatening hemorrhage and necrosis, particularly in the brain (cerebral malaria)
Furthermore, extensive hemolysis and kidney damage occur, with resulting hemoglobinuria.
The dark color of the patients urine has given rise to the term blackwater fever.

Your Answer. a
Correct Answer. c

Copyright 2014 Delhi Academy of Medical Sciences, All Rights Reserved. 51/80
(126). EB virus belongs to which class of virus?

a. Retro virus

b. Herpes virus

c. RNA virus

d. Pox virus

Solution. Ans-126: (b) Herpes virus Ref.: Read the text below Sol :
The EpsteinBarr virus (EBV), also called human herpesvirus 4 (HHV-4), is one of eight viruses in the herpes family, and is one of the
most common viruses in humans.
It is best known as the cause of infectious mononucleosis (glandular fever). It is also associated with particular forms of cancer, such as
Hodgkin's lymphoma, Burkitt's lymphoma, nasopharyngeal carcinoma, and conditions associated with human immunodeficiency virus
(HIV), such as hairy leukoplakia and central nervous system lymphomas

Your Answer. b
Correct Answer. b

(127). The virulence of Pfeiffers bacillus is due to :

a. Potent endotoxin

b. Pili

c. A rubital and a ributol containing capsule

d. Exotoxin

Solution. Ans-127: (c) A rubital and a ributol containing capsule Ref.: Read the text below Sol : Pfeiffers Bacillus is the other name for
Haemophilus influenzae

Your Answer. d
Correct Answer. c

Copyright 2014 Delhi Academy of Medical Sciences, All Rights Reserved. 52/80
(128). Tellurite stimulates growth of :

a. E.coli

b. Cl.tetani

c. C. diphtheria

d. Salmonella

Solution. Ans 128: (c) C. diphtheria Reference Read the text below Sol:
In the specific laboratory media recommended for the cultivation of C. diphtheriae, tellurite, colistin, or nalidixic acid is responsible for
selective isolation of the organism in the presence of other autochthonous pharyngeal microbes.

Your Answer. a
Correct Answer. c

(129). Which one of the following is a dimorphic fungus ?

a. Histoplasma capsulatum

b. Candida albicans

c. Aspergillus fumigatus

d. Pneumocystis carinii

Solution. Ans-129: (a) Histoplasm capsulatum Ref.: Ananthanarayan- 6th Ed. Sol : Dimorphic Fungi
Can exist as both multicellular fungi (molds) and yeasts
Dimorphism in pathogenic fungi typically depends on temperature :
At 37 C : Yeast form.
At 25 C: Mold form.
Systemic dimorphic fungi
Histoplasma capsulatum
Blastomyces dermatitidis
Coccidiodes immitis
Paracoccidiodes brasiliensis
Subcutaneous dimorphic fungi
Sporothrix schenkii
Opportunistic dimorphic fungi
Penicilliosis marneffei

Your Answer. a
Correct Answer. a

(130). A patient with a peptic ulcer advised gastric biopsy. The tissue was cultured on chocolate agar incubated in a microaerophilic
environment at 37C for 5 to 7 days. At 5 days of incubation, colonies appeared on the plate and were curved, Gram-negative rods,
oxidase-positive. Which of the following is the most likely identity of this organism?

a. Campylobacter jejuni

b. Vibrio parahaemolyticus

c. Haemophilus influenzae

d. Helicobacter pylori

Solution. Ans 130: (d) Helicobacter pylori Ref Read the text below Sol:
Helicobacter pylori was first recognized as a possible cause of gastritis and peptic ulcer by Marshall and Warren in 1984.
This organism is readily isolated from gastric biopsies but not from stomach contents.
It is similar to Campylobacter species and grows on chocolate agar at 37C in the same microaerophilic environment suitable for C.
jejuni (Campy-Pak or anaerobic jar [Gas Pak] without the catalyst).
H. pylori, however, grows more slowly than C. jejuni, requiring 5 to 7 days incubation. C. jejuni grows optimally at 42C, not 37C, as
does H. pylori

Your Answer. d
Correct Answer. d

Copyright 2014 Delhi Academy of Medical Sciences, All Rights Reserved. 53/80
(131). The amounts of protein precipitated in a series of tubes containing a constant amount of antibody and varying amounts of antigen are
presented below. In which tube is antigen-antibody equivalence obtained?

a. A

b. B

c. C

d. D

Solution. ans 131 (d)

Ref Read the text below Sol:


In tube d in the question presented, the maximum protein precipitate is observed.
According to the rules governing precipitin reactions, maximum precipitation occurs at approximately antigen-antibody equivalence.
In tubes a through c, antibody excess occurs.

Your Answer. c
Correct Answer. d

(132). There are at least 10 properties of cytokines. Which of the following is one of these characteristics?

a. Mitogenesis

b. B-cell lipids

c. Lipopolysaccharide (LPS) activation

d. T-cell differentiation

Solution. Ans 132 : (a) Mitogenesis Ref Read the text below Sol:
There are at least 10 functions of the known cytokines, including mitogenesis, lymphocyte activation, pyrogenesis, lymphocyte
proliferation, and T-cell differentiation.
Not all cytokines (such as interleukins) are responsible for all of these functions. Cytokines do not activate LPS, but LPS may activate
macrophages.

Your Answer. c
Correct Answer. a

Copyright 2014 Delhi Academy of Medical Sciences, All Rights Reserved. 54/80
(133). Elevated IgG and IgM antibody titers to parvovirus suggest which of the following diagnosis?

a. Fifth disease

b. Susceptibility to chicken pox

c. Possible subacute sclerosing panencephalitis (SSPE)

d. Possible hepatitis B infection

Solution. Ans 133 : (a) Fifth disease Ref Read the text below Sol:
Fifth disease is a viral exanthem commonly seen in children 8 to 12 years old. Children are ill for a few days but recover without
incident.
Unfortunately, if a pregnant female acquires the disease in the first trimester of pregnancy, the fetus is at risk.
The causative agent is thought to be a parvovirus (parvovirus B 19).

Your Answer. a
Correct Answer. a

(134). Which of the following tests depends on the presence of protein A on certain strains of Staphylococcus aureus?

a. Enzyme-linked immunosorbent assay (ELISA)

b. Enzyme multiplied immunoassay test (EMIT)

c. Counterimmunoelectrophoresis (CIE)

d. Coagglutination (COA

Solution. Ans 134: (d) Coagglutination (COA Ref Read the text below Sol:
Of the many methods available for antigen and antibody detection, LA, ELISA, EMIT, CIE, and COA are the most widely used.
Coagglutination (COA), also an agglutination test, is slightly less sensitive than LA but less susceptible to changes in environment (e.g.,
temperature).
Most strains of coagulase-positive staphylococci have protein A in their cell wall. Protein A binds the Fc fragment of microbial antigens in
body fluids.
COA has also been used to rapidly type or group bacterial isolates.

Your Answer. a
Correct Answer. d

(135). Antistreptolysin titer of 400 international units (IU) indicates which one of the following diseases?

a. Chronic infectious mononucleosis

b. Primary syphilis

c. Scarlet fever

d. Primary atypical pneumonia

Solution. Ans 135: (c) Scarlet fever Ref Read the text below Sol:
Scarlet fever is usually a clinical diagnosis subsequent to streptococcal pharyngitis. Acute group
A streptococcal infections result in elevated antibody titers to streptolysin (ASO), DNase B, NADase, and hyaluronidase, all soluble
products of streptococcal growth.
Rubella immune status tests are usually done by hemagglutination inhibition (HI), enzyme immunoassay (EIA), or latex agglutination.
Reactive HI titers are 1:10 or greater.
IgM tests for rubella are often necessary in congenital infection in order to separate maternal from fetal antibodies.

Your Answer. d
Correct Answer. c

Copyright 2014 Delhi Academy of Medical Sciences, All Rights Reserved. 55/80
(136). There has been much speculation on the pathogenesis of group B streptococcal disease in the neonate. Which of the following is the most
likely pathogenic mechanisms?

a. Complement C5a, a potent chemoattractant, activates PMNs

b. The streptococci are resistant to penicillin

c. The alternative complement pathway is activated

d. In the absence of specific antibody, opsonization, phagocyte recognition, and killing do not proceed normally

Solution. Ans 136 : (d) In the absence of specific antibody, opsonization, phagocyte recognition, and killing do not proceed normally Ref
Read the text below Sol:
The incidence of group B streptococcal disease (GBS) is 13 cases per 1000 births. Neonates acquire the disease during birth from
mothers who harbor the organism.
There has been speculation concerning the pathogenesis of GBS. These include failure to activate complement pathways and
immobilization of polymorphonuclear leukocytes (PMNs) due to the inactivation of complement C5A, a potent chemoattractant.
While GBS is relatively more resistant to penicillin than group A streptococci, the great majority of GBS isolates are still penicillin-
susceptible. An aminoglycoside such as gentamicin may be added to GBS treatment regimens due to the relative reduced susceptibility of
some strains

Your Answer. a
Correct Answer. d

(137). A man who has a penile chancre. The VDRL test is negative. Which of the following is the most appropriate course of action?

a. Send the patient home untreated

b. Repeat the VDRL test in 10 days

c. Perform dark-field microscopy for treponemes

d. Swab the chancre and culture on Thayer-Martin agar

Solution. Ans 137 : (c) Perform dark-field microscopy for treponemes Ref Read the text below Sol:
In men, the appearance of a hard chancre on the penis characteristically indicates syphilis.
Even though the chancre does not appear until the infection is 2 or more weeks old, the VDRL test for syphilis still can be negative
despite the presence of a chancre (the VDRL test may not become positive for 2 or 3 weeks after initial infection).
However, a lesion suspected of being a primary syphilitic ulcer should be examined by dark-field microscopy, which can reveal motile
treponemes.

Your Answer. c
Correct Answer. c

(138). A hyperemic edema of the larynx and epiglottis that rapidly leads to respiratory obstruction in young children is most likely to be caused
by which of the following?

a. K. pneumonia

b. M. pneumoniae

c. Neisseria meningitides

d. H. influenzae

Solution. Ans 138: (d) H. influenzae Ref Read the text below Sol:
Haemophilus influenzae is a Gram-negative bacillus. In young children, it can cause pneumonitis, sinusitis, otitis, and meningitis.
Occasionally, it produces a fulminative laryngotracheitis with such severe swelling of the epiglottis that tracheostomy becomes
necessary.
Clinical infections with this organism after the age of 3 years are less frequent.

Your Answer. c
Correct Answer. d

Copyright 2014 Delhi Academy of Medical Sciences, All Rights Reserved. 56/80
(139). In the diarrhoea associated with cholera toxin, there is activation of which of the following enzyme systems?

a. Adenylate cyclase.

b. ATP

c. Guanylate cyclase.

d. Na-glucose co-transporter

Solution. Ans 139: (a) Adenylate cyclase Ref Read the text below Sol:
Cholera toxin has two parts, A and B. B binds while A activates G protein, which activates adenylate cyclase.
Elevated CAMP results in unrestricted chloride secretion from villous crypts.

Your Answer. a
Correct Answer. a

(140). An 80-year-old female presents with confusion associated with a chest infection. She received standard treatment, and four days later
she developed green, then bloody diarrhoea. Which of the following organisms is most likely to be responsible for her diarrhoea?

a. Campylobacter jejuni

b. Clostridium difficile

c. Escherichia coli 0157

d. Methicillin-resistant Staphylococcus aureus

Solution. Ans 140: (b) Clostridium difficile Ref Read the text below Sol:
This is typical of Clostridium infection with pseudomembranous colitis induced by prior treatment with broad spectrum antibiotics such
as cefuroxime, augmentin and the macrolides.
It is treated with oral vancomycin/metronidazole

Your Answer. d
Correct Answer. b

(141). A 28 year old male presents with a four day history of profuse bloody diarrhoea after returning from a holiday in the Far East. Which of
the following regarding his illness is true?

a. Cysts to E. histolytica in the stools confirms a diagnosis of acute amoebic dysentry

b. Sholera is a likely diagnosis

c. Giardiasis is a likely diagnosis

d. Shigellosis is a likely diagnosis

Solution. Ans 141: (d) Shigellosis is a likely diagnosis Ref Read the text below Sol:
Shigellosis is a possible cause of profuse bloody diarrhoea as cholera and giardiasis are associated with watery diarrhoea.
Trophozoites seen in acute amoebic dysentry, and the test is not 100% sensitive.

Your Answer. d
Correct Answer. d

Copyright 2014 Delhi Academy of Medical Sciences, All Rights Reserved. 57/80
(142). Thayer martin is selective medium for :

a. Neisseria gonorroheae

b. Streptococcus pyogenes

c. Streptococcus agalactiae

d. Streptococcus pneumoniae

Solution. Ans-142: (a) Neisseria gonorroheae Ref.: Read the text below Sol :
Thayer-Martin agar (or Thayer-Martin medium) is a Mueller-Hinton agar with 5% chocolate sheep blood and antibiotics.
It is used for culturing and primarily isolating pathogenic Neisseria bacteria, icluding Neisseria gonorrhoeae and Neisseria meningitidis,
as the medium inhibits the growth of most other microorganisms.
When growing Neisseria meningitidis, one usually starts with a normally sterile body fluid (blood or CSF), so a plain chocolate agar is
used.

Your Answer. d
Correct Answer. a

(143). Which of the following substances stimulates interleukin (IL)-2 receptor production on the surface of T cells?

a. IL-1

b. IL-2

c. IL-3

d. IL-4

Solution. Ans-143: (a) IL-1 Ref: Read the text below Sol :
The production of interleukin (IL)-1 by macrophages stimulates production of IL-2 by the targeted T cells as well as the production of IL-
2 receptors on these T cells.

Your Answer. a
Correct Answer. a

(144). Which of the following is invariably a part of the T-cell receptor complex?

a. CD2

b. CD3

c. CD4

d. CD8

Solution. Ans-144: (b) CD3 Ref: Read the text below Sol :
CD3 is invariably part of the T-cell receptor complex. The other constituent is CD4 or CD3.

Your Answer. b
Correct Answer. b

Copyright 2014 Delhi Academy of Medical Sciences, All Rights Reserved. 58/80
(145). Which of the following autoimmune diseases is associated with the presence of autoantibody to the thyroid-stimulating hormone
receptor?

a. Pernicious anemia

b. Congenital agammaglobulinemia

c. Wiskott-Aldrich syndrome

d. Graves' disease

Solution. Ans-145: (d) Graves' disease Ref: Read the text below Sol :
Graves' disease (hyperthyroidism) is characterized by autoantibodies to the thyroid-stimulating hormone receptor and infiltration of the
thyroid gland with T cells and B cells.

Your Answer. d
Correct Answer. d

(146). What type of hypersensitivity reaction is responsible for myasthenia gravis?

a. Type I (anaphylactic) hypersensitivity reaction

b. Type II (cytotoxic) hypersensitivity reaction

c. Type III (immune complex) hypersensitivity reaction

d. Type IV (delayed) hypersensitivity reaction

Solution. Ans-146: (b) Type II (cytotoxic) hypersensitivity reaction Ref: Read the text below Sol :
Myasthenia gravis is characterized by antibody directed to muscle acetylcholine receptors and is thus a class II (cytotoxic)
hypersensitivity reaction.

Your Answer. b
Correct Answer. b

(147). All of the following statements about non spore-forming anaerobes are correct except

a. They are pleomorphic and can be either gram-negative or gram-positive.

b. They are a common cause of intra-abdominal infections.

c. They usually are found in mixed infections.

d. They represent a minority of the total fecal flora.

Solution. Ans-147: (d) They represent a minority of the total fecal flora. Ref: Read the text below Sol :
The non spore-forming anaerobes are present to the extent of 1011 organisms/g of stool, comprising approximately 99% of the normal
fecal flora.

Your Answer. a
Correct Answer. d

Copyright 2014 Delhi Academy of Medical Sciences, All Rights Reserved. 59/80
(148). Thermotomyces is used in microbiology for which of the following?

a. Sterilization standardization

b. Gram staining

c. Selective media preparation

d. Antibiotic sensitivity

Solution. Ans 148: (a) Sterilization standardization Ref Read the text below Sol:
For determining the efficiency of moist-heat sterilization Spores of bacillus stearothermophilus.
For determining the efficiency of dry-heat sterilization Spores of non-toxigenic strains of clostridium tetani

Your Answer. a
Correct Answer. a

(149). Bordetella bronchitis can best be detected by which of the following procedures?

a. Culture of respiratory secretions on Regan-Lowe agar

b. Cold agglutinin test

c. Direct microscopy of sputum by Gram stain

d. Culture of respiratory secretions in HeLa cells after centrifugation of the inoculated tubes

Solution. Ans 149: (a) Culture of respiratory secretions on Regan-Lowe agar Ref Read the text below Sol:
During the winter months, Bordetella infection may be quite prevalent, particularly in those patients whose immunizations are not
current.
Adult Bordetella infection may not present with typical whooping cough symptoms and must be differentiated from other forms of acute
bronchitis by culture on specific media or direct fluorescent microscopy.

Your Answer. d
Correct Answer. a

(150). Fiber-optic scopes are sterilized by:

a. Glutaraldehyde

b. Ethylene oxide

c. Autoclaving

d. Alcohol

Solution. Ans 150: (a) Glutaraldehyde Ref Read the text below Sol: Glutaraldehyde
Its action is similar to formaldehyde.
It is less toxic and irritant to eyes and skin than formaldehyde.
It is specifically effective against tubercle bacilli, fungi and viruses.
It can be safely used to sterilize:- 1. Scopes (Cystoscopes, bronchoscopes, etc) no adverse effect on cement or lenses of these
instruments. 2. Metal instruments. 3. Corrugated rubber tubes. 4. Face masks. 5. Plastic ETTs

Your Answer. a
Correct Answer. a

Copyright 2014 Delhi Academy of Medical Sciences, All Rights Reserved. 60/80
(151). Which one of the following is transmitted by the bite of a hard Ixodes tick?

a. B. (Rochalimaea) henselae

b. E. chaffeensis

c. C. trachomatis

d. R. rickettsii

Solution. Ans 151: (b) E. chaffeensis Ref Read the text below Sol:
Ehrlichia is an obligate, intracellular parasite that resembles rickettsia. E. chaffeensis has been linked to human ehrlichiosis, although
this infection is primarily seen in animals.
The majority of patients with this disease report exposure to ticks.
It is thought that I. scapularis carries Ehrlichia, although the Lone Star tick, A. americanum, may also transmit the disease.

Your Answer. d
Correct Answer. b

(152). Mesophilic organisms are those that grow best at temperature of:

a. -20c to -7c

b. -7c to 20c

c. 25cto 40c

d. 55c to 80c

Solution. Ans 152: (c) 25c to 40c Ref Read the text below Sol:
Bacteria that grow best at temp. 25c to 40c are called mesophilic organisms.
Psychrophilic bacteria grow best at temperature below 20c.
Thermophiles grow best at temp. 55c-80c.

Your Answer. b
Correct Answer. c

(153). N. gonorrhoeae is a fastidious pathogen and found in sites often contaminated with normal flora. Which of the following is the best
medium for isolation?

a. Sheep blood agar

b. Lffler's medium

c. Thayer-Martin agar

d. Thiosulfate citrate bile salts sucrose medium

Solution. Ans 153: (c) Thayer-Martin agar Ref Read the text below Sol:
The medium of choice for the isolation of pathogenic neisseriae is Thayer-Martin (TM) agar.
TM agar is both a selective and an enriched medium; it contains hemoglobin, the supplement Isovitalex, and the antibiotics vancomycin,
colistin, nystatin, and trimethoprim

Your Answer. d
Correct Answer. c

Copyright 2014 Delhi Academy of Medical Sciences, All Rights Reserved. 61/80
(154). What is the most efficient and reliable diagnostic test for RSV ?

a. Immunofluorescent or enzyme staining

b. Sweat chloride test

c. Complete blood count

d. Routine culture

Solution. Ans- 154: (a) Immunofluorescent or enzyme staining Ref.: Read the text below Sol :
The most efficient and expeditious means of diagnosing RSV is by obtaining a nasopharyngeal or throat swab sample for
immunofluorescent or enzyme staining.
It is 90 percent sensitive and specific for RSV infection.
However, the standard of diagnosis remains with a routine culture, which can take 3 to 7 days. Routine lab tests are rarely helpful and
are nonspecific.
For example, a CBC typically shows a normal or elevated white and the differential can be shifted right or left.

Your Answer. c
Correct Answer. a

(155). Passage of the larva to the lungs is the life cycle seen in:

a. Strongoloides stercoralis

b. Ankylostoma duodenale

c. Ascaris lumbricoids

d. Shistosomiasis

Solution. Ans-155: (c) Ascaris lumbricoids Ref.: Read the text below Sol : Ascaris Lumbricoids
Largest nematode parasitizing man.
Also called Common Roundworm
Habitat is lumen of small intestine.
First appearance of eggs in stools is 6070 days. In larval ascariasis, symptoms occur 416 days after infection. The final symptoms are
gastrointestinal discomfort, colic and vomiting, fever, and observation of live worms in stools. Some patients may have pulmonary
symptoms or neurological disorders during migration of the larvae. There are generally few or no symptoms. A bolus of worms may
obstruct the intestine; migrating larvae may cause pneumonitis and eosinophilia. Adult worms have a life-span of 12 years which means
that individuals may be infected all their lives as worms die and new worms are acquired

Your Answer. a
Correct Answer. c

(156). Accole forms are seen in:

a. P. vivax

b. P. falciparam

c. P. malariae

d. P. ovale

Solution. Ans-156: (b) P. falciparam Ref.: Read the text below Sol :
Trophozoites of P falciparum can be found on the edge of the red blood cells. These are known as accole forms and are found as three
distinct types.
COMMON :
The sinel chromatin bead lies on the edge of the cell with most of the cytoplasm extended along the edge on both sides of the bead.
RIM :
The complete parasite lies in a thickened line along the edge of the cell with no evidence of ring formation.
DISPLACED :
The parasites are displaced beyond the edge of the host cell. All degrees of displacement may occur, from partial to marked displacement
with most of the parasite lying beyond the cell margin.

Your Answer. b
Correct Answer. b

Copyright 2014 Delhi Academy of Medical Sciences, All Rights Reserved. 62/80
(157). Which of the following is not true regarding enterococcus?

a. Common species are E. faecalis and E. faecium

b. It is a causes for peritonitis

c. It is universally susceptible to penicillins

d. Can cause intra-abdominal abscess

Solution. Ans-157: (c) It is universally susceptible to penicillins Ref.: Read the text below Sol :
Enterococci are resistant to penicillin
Disease caused by Enterococci
UTI (particularly who are on antibiotic treatment)
Bacterial endocarditis
Liver abscess, intra-abdominal abscess
Surgical wound infection

Your Answer. c
Correct Answer. c

(158). All the following anaerobes are found in normal humans except:

a. Propionbactor organisms

b. Bacteroids

c. Pseudomonads

d. Lactobacillus

Solution. Ans-158: (c) Pseudomonads Ref.: Read the text below Sol :
The anaerobic Gram-positive cocci and anaerobic Gram-positive non-sporeforming rods such as Anaerobic+bacteria, Bifidobacterium,
Eubacterium, Lactobacillus and Propionibacterium species are, for the most part, components of the normal flora of the mucosal surfaces
and, to a lesser extent, the skin.
They are generally considered to be of relatively low virulence and are especially prevalent in infections associated with predisposing or
underlying conditions such as previous surgery, malignancy, immunodeficiency, diabetes and presence of foreign bodies.

Your Answer. c
Correct Answer. c

(159). The commonest organism causing cellulitis is

a. Streptococcus pyogenes

b. Streptococcus faecalis

c. Streptococcus viridans

d. Microaerophilic streptococci

Solution. Ans-159: (a) Streptococcus pyogenes Ref.: Read the text below Sol : Cellulitis :
Diffuse spreading infection of skin (dermis and subcutaneous tissue) especially of lower leg.
Caused by strep pyogenes (MC), Staph, CI perfringens. E. coli.
Major portal of entry for lower leg cellulitis is toe web tinea pedis with fissuring of skin.
Skin become peud orange in appearance; recurrent attack may sometimes affect lymphatic vessels producing permanent swelling called
as solid edema.
Streptococcus cellulitis tends to develop at sites where lymphatic drainage is disrupted.

Your Answer. a
Correct Answer. a

Copyright 2014 Delhi Academy of Medical Sciences, All Rights Reserved. 63/80
(160). Causes of antigenic drift in influenza viral infections:

a. Small mutation in neuraminidase and hemagglutinin

b. Large mutations in hemagglutinin only

c. Step mutations in viral genome

d. None of the above.

Solution. Ans-160: (a) Small mutation in neuraminidase and hemagglutinin Ref.: Read the text below Sol :
Influenza virus type A strains can be classified into subtypes based on variations in their surface antigens.
Originally only variations in the hemagglutinin were studied, and sub typing depended only on antigenic shifts occurring in the
hemagglutinin.
The earliest isolates (WS, PR8 and related strains) were designated A0. In 1946, the hemagglutinin underwent a major change and these
strains (CAM, FM1 and others) were named A1 or A (A prime).
In 1957, new pandemic strains originated in Asia. These were called the A2 (Asian strains.
The next major change occurred in 1968 with the emergence of the A2 (Hong Kong) subtype.
Within each of these subtypes, the strains have shown a gradual antigenic drift.

Your Answer. a
Correct Answer. a

(161). What is the most effective, current, and available therapy for treatment of severe RSV infections in children with congenital
cardiopulmonary anomalies ?

a. Vaccine (respiratory syncytial virus PFP)

b. Ribavirin (hyperimmune RSV immunoglobulin G)

c. Interferon alfa-2a

d. RSIVIG (RSV-neutralizing antibody/respiratory syncytial immunoglobulin)

e. Maternal passive immunization

Solution. Ans- 161: (b) Ribavirin (hyperimmune RSV immunoglobulin G) Ref.: Read the text below Sol :
Aerosolized Ribavirin has been proven safe and effective in over 100,000 clinical test cases.
It is expensive and should be reserved for and routinely administered to infants and children with underlying cardiopulmonary
anomalies who are currently infected with RSV, RSIVIG (RSV-neutralizing antibody/respiratory syncytial immunoglobulin) is currently
being used, but its benefits are still being tested.
However, its use as prophylaxis for high-risk patients has proven beneficial. The RSV vaccine is currently being tested in children in
phases of decreasing chronological age and offers hope in the future for all infants and children.
Alpha-2A-interferon has been disappointing overall; however, administration in an aerosolized form is currently being tested.

Your Answer. d
Correct Answer. b

Copyright 2014 Delhi Academy of Medical Sciences, All Rights Reserved. 64/80
(162). Defect seen in Di George syndrome :

a. Humoral immunity

b. Cell mediated immunity

c. Thymic hyperplasia

d. Autoimmune hemolytic anemia.

Solution. Ans-162: (b) Cell mediated immunity Ref.: Ananthanarayans Microbiology, 8th ed, p-157 Sol : DiGeorge syndrome is a rare
congenital (i.e. present at birth) disease whose symptoms vary greatly between individuals but commonly include a history of recurrent
infection, heart defects, and characteristic facial features. DiGeorge syndrome is caused by a large deletion from chromosome 22,
produced by an error in recombination at meiosis (the process that creates germ cells and ensures genetic variation in the offspring).
This deletion means that several genes from this region are not present in DiGeorge syndrome patients. It appears that the variation in
the symptoms of the disease is related to the amount of genetic material lost in the chromosomal deletion. Although researchers now
know that the DGS gene is required for the normal development of the thymus and related glands, counteracting the loss of DGS is
difficult. Some effects, for example the cardiac problems and some of the speech impairments, can be treated either surgically or
therapeutically, but the loss of immune system T-cells (produced by the thymus) is more challenging.

Your Answer. a
Correct Answer. b

(163). Pneumatocoele is most commonly caused by:

a. Streptococcus pneumonia

b. Klebsiella

c. Hemophilus influenze

d. Staphylococcus aureus

Solution. Ans-163: (d) Staphylococcus aureus Ref.: Internet resources Sol :


Inflammatory pneumatocoeles result from a rapidly progressive inflammation in which there are plugging of the smaller bronchi in the
affected areas, destruction of the distal alveolar tissue, and cystic hyperexpansion of the air space.
Pneumatocoeles are found much more often in children than in adults.
They are seen particularly in infants and children with staphylococcal pneumonia.
However, they have also been described in pneumonia due to other micro-organisms such as Streptococcus pneumonia, Klebsiella,
Hemophilus influenzae, and in measles pneumonitis.
On chest radiographs, they appear as thin-walled, rounded radiolucencies in areas of pneumonic consolidation.

Your Answer. c
Correct Answer. d

(164). Bacteria most commonly involved in prosthetic valvular heart disease within 2 months of surgery is :

a. Staphylococcus epidermidis

b. Enterococci

c. Streptococcus viridians

d. Hemophilus

Solution. Ans-164: (a) Staphylococcus epidermidis Ref.: Ananthanarayans Microbiology, 8th ed, p-202 Sol :
Staphylococci account for 20 to 30% of all cases of endocarditis in the elderly.
The predominant species, staphylococcus aureus, often causes nosocomial endocarditis
S. epidermidis is isolated in < 5% of patients with native valve endocarditis, but it is the most common cause of cases that involve
prosthetic valves occurring early after surgery in elderly and in younger patients.
Gram-negative aerobic bacilli and fungi cause 2 to 3% of all cases of endocarditis.
Infections due to Bacteroides sp and mixed infections are rare among the elderly.

Your Answer. a
Correct Answer. a

Copyright 2014 Delhi Academy of Medical Sciences, All Rights Reserved. 65/80
(165). Which of the following is commonly responsible for toxic shock syndrome in female patient :

a. Streptococcus-group B

b. Pseudomonas

c. H.influenzae

d. Staphylococcus aureus

Solution. Ans-165: (d) Staphylococcus aureus Ref.: Ananthanarayans Microbiology, 8th ed, p-119 Sol :
Toxic shock syndrome is a severe systemic illness characterized by shock, pyrexia, an erythematous rash, gastrointestinal disturbance
and central nervous system signs including lethargy or irritability.
It is mediated by toxins produced by some strains of bacteria, most commonly staphylococcus aureus or Group A streptococcus.
It has a high associated mortality of up to 50% if untreated.
Children under 4 years of age with skin loss are particularly at risk, having not developed antibodies to the toxins produced by the
bacteria.

Your Answer. d
Correct Answer. d

(166). Donovan bodies refers to :

a. Large mononuclear cells with gram-negative rods within the cytoplasm

b. Large mononuclear cells with gram- positive rods within the cytoplasm

c. Small mononuclear cells with gram negative rods within the cytoplasm

d. Small mononuclear cells with gram positive rods within the cytoplasm

Solution. Ans-166: (a) Large mononuclear cells with gram-negative rods within the cytoplasm Ref:Bailey and Loves Short Practice of
Surgery -- 1402 Sol: Donovan bodies short gram negative rods within the cytoplasm of the large mononuclear cells.
Up to 20 intracytoplasmic vacuoles contain pleomorphic donovan bodies in either young uncapsulated form (safety pin appearance) or
mature encapsulated forms.

Your Answer. a
Correct Answer. a

(167). The following cause pulmonary hypertension due to direct involvement of pulmonary vasculature except

a. Sarcoidosis

b. Besnier-Boeck disease

c. Schistosomiasis

d. HIV

Solution. Ans-167: (d) HIV Ref: Read the text below Sol:
Besnier Boeck disease is the other name of sarcoidosis.
Both saracoidosis and Schistosomiasis cause pulmonary hypertension due to direct involvement of pulmonary vasculature.
HIV causes pulmonary hypertension by undefined mechanism.

Your Answer. d
Correct Answer. d

Copyright 2014 Delhi Academy of Medical Sciences, All Rights Reserved. 66/80
(168). Allergic bronchopulmonary aspergillosis [ABPA] is associated with all except

a. Asthma

b. Pulmonary infiltrates

c. Positive culture of A fumigates

d. Serum precipitins

Solution. Ans-168: (c) Positive culture of A fumigates Ref: Harrisons- 197 Sol :
Culture is important in confirming the diagnosis, given that multiple other (rarer) fungi can mimic Aspergillus spp. histologically.
Bacterial agar is less sensitive than fungal media for culture.

Your Answer. d
Correct Answer. c

(169). Hb- H disease, false is

a. More common in Africa

b. 3 alleles of alpha-globin chain missing

c. Anaemia due to unstable haemoglobin

d. Haemoglobin precipitates in circulating RBCs

Solution. Ans-169: (a) More common in Africa Ref: Harrisons- 99 Sol :


HbH: 3 of the 4 alpha globin loci are deleted
Alpha thalassaemia-2 trait is an asymptomatic, silent carrier state.
Alpha-thalassaemia-1 trait resembles beta-thalassaemia minor.
Offspring doubly heterozygous for alpha-thalassaemia-2 and alpha-thalassaemia-1 exhibit a more severe phenotype called HbH disease.
Heterozygosity for a deletion that remove both the genes from the same chromosome is common in Asians.

Your Answer. c
Correct Answer. a

(170). Syphilis doesnt transmit from the child to the mother, but transmits to other children. This is

a. Colles law

b. Kassowitzs law

c. Dideys law

d. Profetas law

Solution. Ans-170: (a) Colles law Ref: Internet Sol :


Colles law states that A child born to a mother who has no signs of venereal symptoms, and presents with this disease at the age of a
few weeks, will infect the healthiest nursemaid, but not its mother.
The observation had been earlier by Simon de Vellember in 1565.

Your Answer. c
Correct Answer. a

Copyright 2014 Delhi Academy of Medical Sciences, All Rights Reserved. 67/80
(171). Main reason for using multiple drugs in TB is

a. To prevent development of resistance

b. To reduce toxicity

c. To effectively kill bacteria

d. To reduce drug dosage

Solution. Ans-171: (a) To prevent development of resistance Ref: Harrisons 17th ed, Ch. 158. Sol :
Chemotherapy for tuberculosis became possible with the discovery of streptomycin in the mid-1940s.
Randomized clinical trials clearly indicated that the administration of streptomycin to patients with chronic tuberculosis reduced
mortality rates and led to the cure in the majority of cases.

Your Answer. a
Correct Answer. a

(172). The signs and symptoms of bacterial meningitis include all the following except

a. Headache severe in character, constant, and in a generalized location

b. A fixed dilated pupil associated with vomiting and fever

c. Drowsiness or coma

d. Irritability

Solution. Ans-172: (b) A fixed dilated pupil associated with vomiting and fever Ref: Read the text below Sol :
A fixed, dilated pupil associated with vomiting and fever is not a sign seen in meningitis. The other choices are seen in bacterial
meningitis.

Your Answer. b
Correct Answer. b

Copyright 2014 Delhi Academy of Medical Sciences, All Rights Reserved. 68/80
(173). A 40 year old farmer presents with lesions on his forearm(shown in image).Colony of the organism on special stain is also shown
below.What is the diagnosis ?

a. Candidiasis

b. Chromoblastomycosis

c. Paracoccidiodomycosis

d. Sporotrichosis

Solution. Ans-173: (d) Sporotrichosis Ref.: Read the text below Sol :
Sporotrichosis is seen in farmers following penetrating injury by plant product(thorn).It is also known as Rose Gardeners disease.
It presents as subcutaneous nodules which may break down to form ulcers.
Cigar shaped yeasts of Sporothrix is shown in the image in question

Your Answer. d
Correct Answer. d

Copyright 2014 Delhi Academy of Medical Sciences, All Rights Reserved. 69/80
(174). About Hepatitis B incorrect statement is

a. Transmission is Mostly vertical than horizontal

b. One month time elapsed between viral marker and symptoms onset

c. Most of the hepatitis transmitted by blood transfusion is not caused by HBV

d. None

Solution. Ans-174: (a) Transmission is Mostly vertical than horizontal. Ref: Jawetz, Melnick, & Adelberg's Medical Microbiology, 24th ed
pg 428, Harrisons internal medicine 17th ed pg 1940. Sol: About transmission of Hepatitis B
Most of the hepatitis transmitted by blood transfusion is not caused by HBV; moreover, in approximately two-thirds of patients with
acute type B hepatitis, no history of an identifiable percutaneous exposure can be elicited. We now recognize that many cases of hepatitis
B result from less obvious modes of nonpercutaneous or covert percutaneous transmission.
HBsAg has been identified in almost every body fluid from infected persons, and at least some of these body fluidsmost notably semen
and salivaare infectious, albeit less so than serum, when administered percutaneously or nonpercutaneously to experimental animals.
Among the nonpercutaneous modes of HBV transmission, oral ingestion has been documented as a potential but inefficient route of
exposure.
By contrast, the two nonpercutaneous routes considered to have the greatest impact are intimate (especially sexual) contact and
perinatal transmission.
According to Jawetz Vertical and contact-associated transmission occurs in endemic regions; parenteral and sexual transmission are the
main modes of transmission in nonendemic regions According to Harrison Percutaneous inoculation has long been recognized as a major
route of hepatitis B transmission About viral markers of hepatitis B
After a person is infected with HBV, the first virologic marker detectable in serum within 112 weeks, usually between 812 weeks, is
HBsAg .
Circulating HBsAg precedes elevations of serum aminotransferase activity and clinical symptoms by 26 weeks and remains detectable
during the entire icteric or symptomatic phase of acute hepatitis B and beyond.
In typical cases, HBsAg becomes undetectable 12 months after the onset of jaundice and rarely persists beyond 6 months.
After HBsAg disappears, antibody to HBsAg (anti-HBs) becomes detectable in serum and remains detectable indefinitely thereafter.
Because HBcAg is intracellular and, when in the serum, sequestered within an HBsAg coat, naked core particles do not circulate in
serum and, therefore,
HBcAg is not detectable routinely in the serum of patients with HBV infection. By contrast, anti-HBc is readily demonstrable in serum,
beginning within the first 12 weeks after the appearance of HBsAg and preceding detectable levels of anti-HBs by weeks to months.
Because variability exists in the time of appearance of anti-HBs after HBV infection, occasionally a gap of several weeks or longer may
separate the disappearance of HBsAg and the appearance of anti-HBs.
During this "gap" or "window" period, anti-HBc may represent the only serologic evidence of current or recent HBV infection, and blood
containing anti-HBc in the absence of HBsAg and anti-HBs has been implicated in the development of transfusion-associated hepatitis
B.Isolated anti-HBc does not necessarily indicate active virus replication; most instances of isolated anti-HBc represent hepatitis B
inection in the remote past.
Rarely, however, isolated anti-HBc represents low-level hepatitis B viremia, with HBsAg below the detection threshold; occasionally,
isolated anti-HBc represents a cross-reacting or false-positive immunologic specificity.
Recent and remote HBV infections can be distinguished by determination of the immunoglobulin class of anti-HBc.
Anti-HBc of the IgM class (IgM anti-HBc) predominates during the first 6 months after acute infection, whereas IgG anti-HBc is the
predominant class of anti-HBc beyond 6 months.
Therefore, patients with current or recent acute hepatitis B, including those in the anti-HBc window, have IgM anti-HBc in their serum.
In patients who have recovered from hepatitis B in the remote past as well as those with chronic HBV infection, anti-HBc is
predominantly of the IgG class.
Infrequently, in <15% of patients with acute HBV infection, levels of HBsAg are too low to be detected; in such cases, the presence of
IgM anti-HBc establishes the diagnosis of acute hepatitis B.
When isolated anti-HBc occurs in the rare patient with chronic hepatitis B whose HBsAg level is below the sensitivity threshold of
contemporary immunoassays (a low-level carrier), the anti-HBc is of the IgG class.
Generally, in persons who have recovered from hepatitis B, anti-HBs and anti-HBc persist indefinitely.

Your Answer. c
Correct Answer. a

Copyright 2014 Delhi Academy of Medical Sciences, All Rights Reserved. 70/80
(175). A man had Egg omelet 1-4 hours back and developed diarrhea, vomiting and abdominal cramps. Most likely cause of poisoning.

a. Staphylocoocus

b. Salmonella

c. Bacillus cereus

d. Clostridium

Solution. Ans-175: (a) Staphylocoocus Ref.: Read the text below Sol :

Your Answer. a
Correct Answer. a

(176). EB virus causes all the following except:

a. Hodgkins lymphoma

b. Kaposis sarcoma

c. Nasopharyngeal CA

d. Burkitts lymphoma

Solution. Ans-176: (b) Kaposis sarcoma Ref.: Read the text below Sol :
Kaposis sarcoma is associated with HIV.
It is best known as the cause of infectious mononucleosis (glandular fever). It is also associated with particular forms of cancer, such as
Hodgkin's lymphoma, Burkitt's lymphoma, nasopharyngeal carcinoma, and conditions associated with human immunodeficiency virus
(HIV), such as hairy leukoplakia and central nervous system lymphomas.
There is evidence that infection with the virus is associated with a higher risk of certain autoimmune diseases,especially
dermatomyositis, systemic lupus erythematosus, rheumatoid arthritis, Sjgren's syndrome, and multiple sclerosis

Your Answer. b
Correct Answer. b

Copyright 2014 Delhi Academy of Medical Sciences, All Rights Reserved. 71/80
(177). All are true about streptococcus except:

a. M-protein is responsible for production of mucoid colonies

b. M- protein is responsible for virulence

c. Mucoid colonies are virulent

d. No resistance to penicillin has been reported

Solution. Ans-177: (a) M-protein is responsible for production of mucoid colonies Ref.: Read the text below Sol :
Mucoid colonies are due to production of capsule of hyaluronic acid not due to M-protein.
Growth characteristic of strep. Pyogenes :
On blood agar virulent stains form matt or mucoid colones while avirulent from glossy colonies. M, T, R are proteins found on outer part
of cell wall and forms the basis of Griffith classification.
M protein acts as virulence factor and antibody against it are protective.
T and R protein No relation to virulence

Your Answer. a
Correct Answer. a

(178). In a case of neonatal meningitis, the etiologic bacteria was found to have properties of hemolysis, bacitracin resistance, CAMP positive.
Which of the following is most likely causative agent?

a. S. pyogenes

b. S. agalactiae

c. S. pneumonia

d. E. faecalis

Solution. Ans-178: (a) S. pyogenes Ref.: Read the text below Sol :
S. agalactiae is the most common cause of neonatal meningitis in west.
It does not hydrolyse bile esculin agar, however hydrolyse sodium hippurate and are bacitracin resistant.
Identified by CAMP (Christie, Atkins and Munch Peterson) reaction.

Your Answer. b
Correct Answer. a

(179). Chlamydia are isolated by :

a. Enzyme immune assay

b. Yolk sac inoculation

c. Direct immunofluorescence

d. Micro immunofluorescence

Solution. Ans-179: (b) Yolk sac inoculation Ref.: Read the text below Sol : Isolation of chlamydiae can done by :
Inoculation into yolk sac/embryonated eggs of 6-8 day old chick embryo which may be pretreated by streptomycin or polymyxin B.
Inculation into experimental animals (mice)
Tissue/cell culture :
Preferred mode
Commonly used cell lines are McCoy and HeLa cells.
Cell cultures are pretreated by irradiation or chemicals such as 5-iodo 2 deoxyuridic or cycloheximide to enhance replication and
detection of inclusion bodies.
Pretreatment with DEAE dextrax or centrifugation after inoculation, promotes contact between chlamydiae particles and cell monolayer.

Your Answer. b
Correct Answer. b

Copyright 2014 Delhi Academy of Medical Sciences, All Rights Reserved. 72/80
(180). Varicella virus remain latent in :

a. T cell

b. B cell

c. Microglia

d. Trigeminal ganglion

Solution. Ans-180: (d) Trigeminal ganglion Ref.: Read the text below Sol : Site of latency of varicella virus
Dorsal root ganglia (Most frequent) T3-L2
Trigeminal ganglia (frequently ophthalmic branch)

Your Answer. d
Correct Answer. d

(181). A 29-year-old person comes with focal seizures. MRI shows frontal and temporal enhancement. What is the most probable diagnosis?

a. Meningococcal meningitis

b. Herpes simplex encephalitis

c. Japanese encephalitis

d. Enterovirus encephalitis

Solution. Ans-181: (b) Herpes simplex encephalitis Ref.: Read the text below Sol :
In children and young adults primary HSV infection can led to encephalitis, by neurotropic spread of virus from periphery via olfactory
bulb.
In other majority of cases, there is prior mucocutaneous HSV-1 infection which gets reactivated.
Clincial hallmark of HSV infection include acute onset of fever, and focal neurological symptoms and signs, especially of temporal lobe.
Diagnosis :
CSF protein and CSF lymphocytosis
Brain biopsy is gold standard
HSV DNA detection in CSF by PCR has largely replaced biopsy
Treatment : IV acyclovir

Your Answer. a
Correct Answer. b

(182). In a asthmatic patient; causative agent was found to be fungi with septate hyphae. The organism is :

a. Aspergillus

b. Candida

c. Mucor

d. Cryptococcus

Solution. Ans-182: (a) Aspergillus Ref.: Read the text below Sol :
A fungi with septate hyphae in asthmatic patient can be none other than aspergillus, and this is a case of allergic bronchopulmonary
aspergillosis

Your Answer. a
Correct Answer. a

Copyright 2014 Delhi Academy of Medical Sciences, All Rights Reserved. 73/80
(183). Superantigen include all, except:

a. TSST

b. Exfoliative toxin

c. Pyrogenic exotoxin

d. Cholera toxin

Solution. Ans-183: (d) Cholera toxin Ref.: Read the text below Sol :
Superantigens are certain protein molecules that activate very large number of T cells irrespective of antigenic specifities
They bind directly to lateral aspect of TCR chain and activate upto 20% of the circulating T cells compared to conventional (0.001%)
causing multisystem dysfunction.
Examples :
Toxic shock syndrome toxin
Exfoliative toxin
Staphylococcal anterotoxin
Pyrogenic exotoxin

Your Answer. d
Correct Answer. d

(184). Characteristic of Bacilus cereus food poisioning is

a. Presence of Fever

b. Presence of Pain abdomen

c. Absence of Vomiting

d. Absence of Diarrhoea

Solution. Ans-184: (b) Presence of Pain abdomen . Ref: Read the text below. Sol:
Abdominal cramps, vomiting and diahorhea are common.
Fever is uncommon as it is toxin mediated food posioing.

Your Answer. b
Correct Answer. b

(185). The leading cause of meningitis in HIV + patients is

a. Cryptococcus

b. Cytomegalovirus

c. Meningococcus

d. Toxoplasmosis

Solution. Ans-185: (a) Cryptococcus Ref: Read the text below. Sol:
Meningitis in patients with HIV infection is almost always infectious in origin. Two opportunistic pathogens stand out as important
problems in patients with AIDS - Cryptococcus neoformans and Mycobacterium tuberculosis, and together they account for about of
the cases of meningitis.
The rates of cryptococcal meningitis have been decreasing in the Western world with the introduction of effective suppressive therapy,
especially fluconazole, and the introduction of effective treatment for HIV itself, with the consequent improvement in immune function.
However, in developing countries, both tuberculosis and cryptococcosis remain among the most common and important causes of
morbidity and mortality in patients with AIDS.

Your Answer. a
Correct Answer. a

Copyright 2014 Delhi Academy of Medical Sciences, All Rights Reserved. 74/80
(186). Organism causing billiary duct obstruction :

a. Clonorchis sinensis

b. Ankylostoma duodenale

c. Strongyloides stercales

d. Enterobius vermicularis

Solution. Ans-186: (a) Clonorchis sinensis Ref.: Read the text below Sol : Hepatobiliary Parasitism
Common in Southern china
Biliary tract is infestated by adult helminthes or their ova
Causes :
Trematodes (Liver or biliary flukes) MC
Clonorchis sinensis
Opisthorchis viverrini or O. felineus
Fasciola hepatica
Nematode (Ascaris lumbricoides by intraductal migration of adult worm

Your Answer. a
Correct Answer. a

(187). True statement about antistreptolysin O titre is :

a. In normal people the titre is < 200

b. In acute glomerulonephritis the titre is low

c. ASO titre > 200 indicate rheumatic fever

d. Streptozyme test is an active hemagglutination test

Solution. Ans-187: (b) In acute glomerulonephritis the titre is low Ref.: Read the text below Sol : ASO (Anti Streptolysin O) titre :
Estimation of antibody against streptolysin is a standard serological test for retrospective diagnosis of streptococcal infection
ASO titre > 200 are indicative of prior streptococcal infection
Raised after throat infection only
In acute glomerulonephritis the titre is low

Your Answer. c
Correct Answer. b

(188). All are true about scrub typhus, except :

a. Causative organism is O. Tsutsugamoshi

b. Vector is trombiculide mite

c. Adult female feeds on vertebrate hosts

d. Tetracycline is the drug of choice

Solution. Ans-188: (c) Adult female feeds on vertebrate hosts Ref.: Read the text below Sol :
Mite feeds on serum of warm blood animals only during there larval stage (chiggers) and adult mites feed only on plants
Scrub typhus :
Caused by O. tsutsugamushi
Transmitted by trombiculid mite which also shows transovarian spread
Clinical features :
Fever, headache, myalgia, cough and GI symptoms.
Classic case includes an eschar, regional lymphadenopathy and a maculopapular rash.

Your Answer. c
Correct Answer. c

Copyright 2014 Delhi Academy of Medical Sciences, All Rights Reserved. 75/80
(189). Which portion of MHC 1 complex forms the component of antigen presenting part :

a. Between alpha 1 beta 2 microglobulin

b. Distal part of alpha chain

c. Proximal part of alpha chain

d. Between aS and b2 microglobulin

Solution. Ans-189: (b) Distal part of alpha chain Ref.: Read the text below Sol : Antigen Presenting part of MHC MHC I/HLAI :
Class 1 molecule consists of a three domains of heavy alpha peptide chain (a1 ; a2; a3) which are non-covalently linked to smaller 2
microglobulin peptide.
The distal domain (alpha 1 and alpha 2) of MCH 1 have highly variable amino acid sequences and are folded to form a cavity or groove.
Protein antigens are bound to this groove for presentation to CD8 T cells.
MHC II/HLA II :
MHC II antigens are heterodimer consisting of an alpha and beta chain. Each chain has two domain proximal one is constant and distal
one is variable.
The two distal domain (alpha 1, beta 1) constitute the antigen binding site, for recognition by CD4 T cells.
Both class 1 and class 2 are members of immunoglobulin gene super family.
MHC III /HLA III
MHC III are heterogenous they include complement components involved in the formation of C3 convertase, heat shock protein and
tumor necrosis factor.

Your Answer. a
Correct Answer. b

(190). Gram-negative bacteria doesnt take gram stain because it is made of:

a. Polysaccharide

b. Lipopolysaccharide

c. Techoic acid

d. None of the above.

Solution. Ans-190: (d) None of the above. Ref.: Read the text below Sol :
The exact mechanism of gram reaction is not known. The possible reason are :
Gram positive bacteria have a more acidic protoplasm, which account for their retaining the basic primary dye more strongly
Dye lodine complex is retained in gram positive cells by thick peptide glycan mesh and is readily wash through the very thin
peptidoglycan layer of gram ve cells.

Your Answer. c
Correct Answer. d

(191). One virus particles prevents multiplication of 2nd virus. This phenomena is:

a. Viral interference

b. Mutation

c. Supervision

d. Permutation

Solution. Ans-191: (a) Viral interference Ref.: Read the text below Sol : Viral interference
Interference in which infection of a cell by one virus inhibits simultaneous or subsequent infection by another virus.
Most important mediator of interference is interferon.
Interference produced by destruction of cell receptors is seen with myxoviruses and enterovirus.
It is applied in the field in controlling poliomyelitis outbreaks by in controlling poliomyelitis outbreaks by introducing into the
population, the live attenuated poliovirus vaccine.

Your Answer. a
Correct Answer. a

Copyright 2014 Delhi Academy of Medical Sciences, All Rights Reserved. 76/80
(192). Treatment of partner is required in all infection except:

a. Candida

b. Herpes

c. Trichomonas

d. Gardnerella

Solution. Ans-192: (d) Gardnerella Ref.: Read the text below Sol :
Treatment of male partners with metronidazole does not prevent recurrence of bacterial vaginosis/gardenella associated with vaginal
discharge.
Treatment of sexual partners is required in :
Candidiasis
Herpes genitalis
Trichomoniasis.

Your Answer. a
Correct Answer. d

(193). Resolution provided by light microscope is:

a. 200 nm

b. 20 nm

c. 0.2 nm

d. 2.0 nm

Solution. Ans-193: (a) 200 nm Ref.: Read the text below Sol : Resolving Power are :
Light microscope 0.25 um 0.3 um 200-300 nm
Electron microscope 2-10 A = 0.2 1 nm

Your Answer. a
Correct Answer. a

(194). Toxin involved in the streptococcal toxic shock syndrome is :

a. Pyrogenic toxin

b. Streptokinase

c. Hemolysin

d. Neurotoxin

Solution. Ans-194: (a) Pyrogenic toxin Ref.: Read the text below Sol : Streptococcal TSS is caused by pyrogenic exotoxin.
Pyrogenic exotoxin = Erythrogeic toxin = Dick = Scarlatinal toxin
Produced by group A streptococci (type 1, 2, 3, 12 and 28)
There are 3 type of pyrogenic exotoxin : A, B, and C
A (MC cause of streptococcal TSS) and C are coded by bacteriophage gene, while B is chromosomal
Acts as superantigen
Associated with streptococcal toxic shock syndrome and scarlet fever

Your Answer. a
Correct Answer. a

Copyright 2014 Delhi Academy of Medical Sciences, All Rights Reserved. 77/80
(195). Streptococci causing dental caries :

a. Streptococci equisimilis

b. Streptococci mutans

c. Streptococci pneumoniae

d. Streptococci bovis

Solution. Ans-195: (b) Streptococci mutans Ref.: Read the text below Sol : S. mutans :
It is member of viridans streptococci (-hemolytic)
It assumes bacillary form in acid environment
Can synthesize acid and large polysaccharide (adhesive dextran or levan) from sucrose.

Your Answer. b
Correct Answer. b

(196). In the gut, anaerobic bacteria outnumber the aerobes by a ratio of :

a. 10:1

b. 100 : 1

c. 1000 : 1

d. 10,000 : 1

Solution. Ans-196: (c) 1000 : 1 Ref.: Read the text below Sol :
Anaerobes outnumber facultative organism by 1000 fold.
Normal flora of intestinal tract
At birth the intestine is sterile, but organism are soon introduced after birth.
In breastfeed children lactic acid streptococci and lactobacilli seen.
Microorganism are minimum (103 105g/ of contents) in stomach.
In upper intestine lactobacilli and enterococci predominate.
In colon Bacterides fragillis is the most common organism found.
Intestinal bacteria are important in synthesis of vitamin K.

Your Answer. d
Correct Answer. c

(197). In all of the following diseases chronic carriers are found except :

a. Measles

b. Typhoid

c. Hepatitis B

d. Gonorrhea

Solution. Ans-197: (a) Measles Ref.: Read the text below Sol : In measles and whooping cough only cases are found with no carriers.
Chronic carriers seen in :
Typhoid
Dysentery
Hepatitis B
Malaria
Cerebrospinal meningitis
Gonorrhea

Your Answer. a
Correct Answer. a

Copyright 2014 Delhi Academy of Medical Sciences, All Rights Reserved. 78/80
(198). A woman with infertility receives an ovary transplant from her sister who is an identical twin. What type of graft is it?

a. Xenograft

b. Autograft

c. Allograft

d. Isograft

Solution. Ans-198: (d) Isograft Ref.: Read the text below Sol :

Your Answer. d
Correct Answer. d

(199). Sputum examination is not useful in diagnosis of :

a. Trichuriasis trichiura

b. Ancylostoma duodenale

c. Paragonimus

d. Strongyloides

Solution. Ans-199: (a) Trichuriasis trichiura Ref.: Read the text below Sol : Any parasite which pass through lungs at some or other
stage during their the cycle, sputum examination can be done for them. These parasites are :
Rhabditiform larva of Ascaris
Golden brown eggs of Paragonimus westermani (Lung fluke)
Filariform larvae of Strongyloides, Ancylostoma
Entamoeba histolytica Chocolate brown sputum due to hepatobronchal fistula.

Your Answer. a
Correct Answer. a

(200). All the following are true about Brugla malayi except :

a. The intermediate hosts in the India are Mansoni

b. The tail tip is free nuclei

c. Nuclei are blurred, so counting is difficult

d. Adult worm is found in the lymphatic system

Solution.
Ans-200: (b) The tail tip is free nuclei Ref.: Read the text below Sol : Microfilaria tail free of nuciei :
W. Bancrofti
O. Volvulus
M. Ozzardi

Your Answer. b
Correct Answer. b

Copyright 2014 Delhi Academy of Medical Sciences, All Rights Reserved. 79/80
Test Answer
1.(c) 2.(a) 3.(d) 4.(d) 5.(a) 6.(c) 7.(c) 8.(b) 9.(a) 10.(a)

11.(a) 12.(c) 13.(b) 14.(a) 15.(c) 16.(c) 17.(a) 18.(d) 19.(c) 20.(b)

21.(b) 22.(c) 23.(d) 24.(c) 25.(c) 26.(b) 27.(c) 28.(c) 29.(a) 30.(d)

31.(b) 32.(b) 33.(d) 34.(b) 35.(a) 36.(a) 37.(b) 38.(d) 39.(d) 40.(b)

41.(d) 42.(d) 43.(b) 44.(c) 45.(a) 46.(a) 47.(b) 48.(c) 49.(b) 50.(c)

51.(a) 52.(b) 53.(a) 54.(a) 55.(b) 56.(c) 57.(b) 58.(b) 59.(b) 60.(d)

61.(d) 62.(c) 63.(c) 64.(c) 65.(c) 66.(c) 67.(b) 68.(c) 69.(c) 70.(b)

71.(c) 72.(d) 73.(d) 74.(b) 75.(a) 76.(d) 77.(b) 78.(d) 79.(c) 80.(b)

81.(d) 82.(d) 83.(d) 84.(c) 85.(c) 86.(c) 87.(d) 88.(b) 89.(d) 90.(a)

91.(b) 92.(d) 93.(c) 94.(c) 95.(d) 96.(b) 97.(d) 98.(b) 99.(c) 100.(a)

101.(a) 102.(c) 103.(b) 104.(b) 105.(b) 106.(a) 107.(a) 108.(c) 109.(c) 110.(d)

111.(d) 112.(b) 113.(c) 114.(a) 115.(a) 116.(d) 117.(b) 118.(c) 119.(b) 120.(c)

121.(b) 122.(c) 123.(b) 124.(b) 125.(c) 126.(b) 127.(c) 128.(c) 129.(a) 130.(d)

131.(d) 132.(a) 133.(a) 134.(d) 135.(c) 136.(d) 137.(c) 138.(d) 139.(a) 140.(b)

141.(d) 142.(a) 143.(a) 144.(b) 145.(d) 146.(b) 147.(d) 148.(a) 149.(a) 150.(a)

151.(b) 152.(c) 153.(c) 154.(a) 155.(c) 156.(b) 157.(c) 158.(c) 159.(a) 160.(a)

161.(b) 162.(b) 163.(d) 164.(a) 165.(d) 166.(a) 167.(d) 168.(c) 169.(a) 170.(a)

171.(a) 172.(b) 173.(d) 174.(a) 175.(a) 176.(b) 177.(a) 178.(a) 179.(b) 180.(d)

181.(b) 182.(a) 183.(d) 184.(b) 185.(a) 186.(a) 187.(b) 188.(c) 189.(b) 190.(d)

191.(a) 192.(d) 193.(a) 194.(a) 195.(b) 196.(c) 197.(a) 198.(d) 199.(a) 200.(b)

Copyright 2014 Delhi Academy of Medical Sciences, All Rights Reserved. 80/80

You might also like